Sei sulla pagina 1di 86

A client with pemphigus is being seen in the clinic regularly.

The nurse plans care based on which of the


following descriptions of this condition?
A. The presence of tiny red vesicles
B. An autoimmune disease that causes blistering in the epidermis
C. The presence of skin vesicles found along the nerve caused by a virus
D. The presence of red, raised papules and large plaques covered by silvery scales

The nurse is caring for a client in the coronary care unit. The display on the cardiac monitor indicates
ventricular fibrillation. What should the nurse do first?
A. Perform defibrillation
B. Administer epinephrine as ordered
C. Assess for presence of pulse
D. Institute CPR
Answer C: Assess for presence of pulse .Artifact can mimic ventricular fibrillation on a cardiac monitor. If the
client is truly in ventricular fibrillation, no pulse will be present.

The nurse is assigned to a newly delivered woman with HIV/AIDS. The student asks the nurse about how it is
determined that a person has AIDS other than a positive HIV test. The nurse responds
A. "The complaints of at least 3 common findings."
B. "The absence of any opportunistic infection."
C. "CD4 lymphocyte count is less than 200."
D. "Developmental delays in children."
Answer C: "CD4 lymphocyte count is less than 200." CD4 lymphocyte counts are normally 600 to 1000.

The home care nurse is performing an assessment on a client who has been diagnosed with an allergy to latex.
In determining the client’s risk factors associated with the allergy, the nurse questions the client about an
allergy to which food item?
A. Eggs
B. Milk
C. Yogurt
D. Bananas
Answer: D

Initial step while detecting pulmonary embolism?


A. Start IV line
B. Check vitals
C. Administer morphine
D. Administer oxygen
Answer: D
Major health complications associated with maternal Zika virus infection?
A. Macrocephaly
B. Microcephaly
C. Rheumatic heart disease
D. Myasthenia gravis .
Answer: B

While assessing a client in an outpatient facility with a panic disorder, the nurse completes a thorough health
history and physical exam. Which finding is most significant for this client?
A. Compulsive behavior
B. Sense of impending doom
C. Fear of flying
D. Predictable episodes
Answer is B: Sense of impending doom.
The feeling of overwhelming and uncontrollable doom is characteristic of a panic attack.

Indications for fundoplication?


A. Hiatus hernia
B. Diaphragmatic hernia
C. GERD
D. All the above
Answer: D

Left ventricular pressure can be measured by using


A. C.V. C
B. Pulmonary artery catheter
C. Swanz - Ganz Catheter
D. All the above
E. Both B&C
Answer: E

United Nations decided to mark the importance of India’s former President and great scientist APJ Abdul Kalam
and declared his birthday as ‘World Students Day’. The world students day is ....?
A. November 15
B. October 15
C. October 17
D. November 17
Answer: B

Sex education should be given to


A. Toddler
B. Preschooler
C. Schoolar
D. Adolescents
Answer: D

Dowager's hump is the forward curvature(kyphosis) of the spine resulting in a stoop, caused by collapse of the
front edges of the thoracic vertebrae commonly seen in....?
A. Osteoarthritis
B. Rheumatoid arthritis
C. Osteoporosis
D. Lumbar fracture
Answer; C

Cessation of breathing more than ....seconds is known as apnea


A. 5
B. 10
C. 15
D. 20
E. 30
Answer: D

While newborns vitals assessments , it should be


A. T,P,R
B. R,P,T
C. P,T,R
D. P,R,T
Answer: B

Koplik spots (also Koplik's sign) are a prodromic viral enanthem of ---- disease?
A. Chikun gunya
B. Diphtheria
C. Herpes zoster
D. Herpes simplex
E. Measles.
Answer: E

CVP is the pressure within


a)inferior venacava
b)pulmonary artery
c)pulmonary vein
d)subclavian vein
Anti - infective vitamin?
A. A
B. B12
C. C
D. D
Answer: A

Scilent killer in neonates?


A. Hypothermia
B hypoxia
C. Hypoglycemia
D. Dehydration
Answer: A

Which of the following condition Christmas disease ?


A. Leukemia
B. DIC
C. Hemophilia A
D. Hemophilia B
Answer : D

Zika is a viral diseases transmitted by mosquito


A. Culex
B. Ades
C. Anopheles
D. Asian tiger
Answer : B

Malnutrition landmark in children?


A. Head circumference
B. Chest circumference
C. Mid arm circumference
D. Milestone achievement
Answer: C

Trendelenburg test is used to detect


A. DVT
B. Varicose vein
C . vulvular disorder
Do thrombophlebitis
A reflex that is seen in normal newborn babies, who automatically turn the face toward the stimulus and make
sucking motions with the mouth when the cheek or lip is touched.
A. Moro
B. Rooting
C. Sucking
D. Swallowing
Answer: B

Rh positive mother has to receive Rh immunoglobulin at


A. 14 weeks
B. 26 weeks
C. 32 weeks
D. None of these.
E. 28 weeks
F. After delivery
Answer: D

CA cervix caused by
A. HPV
B. H. Pylori
C. E coli
D. Treponema
E. Gardnerella vaginalis
Answer: A

" Vande matharam " project associated with


A. Immunization
B. Antenatal care
C. Breast feeding
D. Child care
Answer: B

Royal disease is....


A. DM
B. Hypertension
C. Hemophilia
D. Nephrotic syndrome
Answer: C

Immediately after amniotomy the nurse should check


A. Uterine tone
B. Bladder distension
C. FHS
D. BP
E. Cervical dilation
Answer: C

Degree 4 th , uterine prolapse?


A. uterine inversion
B. Uterine atony
C. Parametritis
D. procidentia
Answer: D

Toxic shock syndrome is due to


A. Streptococcus
B. Staphylococcus aureus.
C. Pneumococus
D. Haemophilus.
Answer: B
Toxic shock syndrome is a rare, life-threatening complication of certain types of bacterial infections. Often toxic
shock syndrome results from toxins produced by Staphylococcus aureus (staph) bacteria, but the condition
may also be caused by toxins produced by group A streptococcus (strep) bacteria.

Molloscum contagiosum is caused by...


A. Poxvirus
B. Candia
C. HPV
D. Variola
E. Gonorrhoea
Answer: A

Colostrum contains highest %


A. Carbohydrates
B. Proteins
C. Fats
D. Vitamins & minerals
Answer: B
Normal sperm densities range
A. 20 - 300 million/ ml
B. 10- 20 billion/ ml
C. 10000- 20000/ ml
D. None of these.
Answer: A

Mr.Ashok orients his staff on the patterns of reporting relationship throughout the organization. Which of the
following principles refer to this?
A.Span of control
B. Hierarchy
C.Esprit d’ corps
D. Unity of direction
Answer: B

Centralized organizations have some advantages. Which of the following statements are TRUE?
A. Highly cost-effective
B. Makes management easier
C. Reflects the interest of the worker
D. Allows quick decisions or actions.
E. Both A&B
F. Both C&D
Answer: E

Which of the following guidelines should be least considered in formulating objectives for nursing care?
A. Written nursing care plan
B. Holistic approach
C. Prescribed standards
D. Staff preferences
Answer: D

Rh negative mother has to receive RH D immunoglobulin with in ----- days postpartum


A. 1
B. 2
C. 3
D. 7
E. None
Answer: C

Pelvic cellulitis
A. Parametritis
B. Vulvitis
C. Pelvic abscess
D. Perinitis
Answer: A

An opioid analgesic is administered to a client during surgery. The nurse assigned to care for the client ensures
that which medication is readily available if respiratory depression occurs?
A. Betamethasone
B. Morphine sulfate
C.Naloxone (Narcan)
D. Meperidine hydrochloride (Demerol)
Answer: C

Fetal bradycardia means HR below


A. 80/mts
B. 100/mts
C. 120/mts
D. 140/mts
Answer: C

A good fetal outcome in contraction test ...?


A. Neutral
B. Positive
C. Negative
D. None of these.
Answer: C

Liquor amnie exceeds 2000ml


A. Hydramnios
B. Oligohydramnions
C. Polyhydramnions
D. Both A & C
Answer : D

MTP act enforced in ...?


A. 1927
B.1972
C. 1989
D. 1871
Answer: B

Early sign of DIC


A. Pain
B. Hematuria
C. Clot formation
D. Vascular obstruction
Answer: B

Highest degree of abortion seen among


A. Husband with A blood group and wife with O group
B. Husband with O blood group and wife with A group
C. Husband with A blood group and wife with B group
D. Husband with AB blood group and wife with O group
Answer: A

.Deferoxamine is administered in overdose of:


A. Iron
B. Calcium gluconate
C. Digoxin
D. Beta blockers
Answer: A

The nurse is preparing to teach a client how to use crutches. Before initiating the lesson, the nurse performs an
assessment on the client.The priority nursing assessment should include which focus?
A. The client's feelings about the restricted mobility
B. The client's fear related to the use of the crutches
C. The client's muscle strength and previous activity level
D. The client's understanding of the need for increased mobility
Answer: C

Most specific enzyme for MI?


A.CPK-M,
B.CPK-MB,
C.CPK-BB,
D.LDH,
Answer: B

Which is the following largest and most muscular chamber of heart


a)right atrium
b)right ventricle
c)left ventricle
d)left atrium

Uterine contractions monitored by ......?


A. Friedman's curve
B. Tonometer
C.Tocodynamo meter
D. Fetoscope
Answer: c

Which of the following drug shows drug holiday


a)Ecospirin
b)streptokinase
c)morphine
d)digoxin

After TURP, the client having continues bladder irrigation. Which of these statements explain the reason for
continuous bladder irrigation?
a. To remove clot from the bladder
b. To maintain the patency of the catheter
c. To maintain the patency of the bladder
d. To dilute urine
Answer: A

The nurse has developed a plan of care for a client diagnosed with anorexia nervosa. Which client problem
should the nurse select as the priority in the plan of care?
A. Malnutrition
B. Inability to cope
C. Concern about body appearance
D. Lack of knowledge about nutrition
Answer: A

Which hormone is responsible for amenorrhoea after delivery or in postpartum period..(in proper lactating
women)..??
A. Oxytocin
B. Prolactin
C. FSH
D. LH
Answer: B

Why should an infant be quiet and seated upright when the nurse checks his fontanels?
A. The mother will have less trouble holding a quiet, upright infant.
B. Lying down can cause the fontanels to recede, making assessment more difficult.
C. The infant can breathe more easily when sitting up.
D. Lying down and crying can cause the fontanels to bulge.
Answer: D
Which of the following is an appropriate nursing diagnosis for a client with renal calculi?
A. Ineffective tissue perfusion
B. Functional urinary incontinence
C. Risk for infection
D. Decreased cardiac output
Answer: C

Most common site of hematoma during postpartum period?


A. Uterine hematoma
B.Vaginal hematoma
C.cervical hematoma
D. Vulvar hematoma
Answer: D

Which trait is the most important for ensuring that a nurse-manager is effective?
A. Communication skills
B. Clinical abilities
C. Health care experience
D. Time management skills
Answer :A
Communication skills are a necessity for a successful nurse-manager. The manager must be able to
communicate with the staff, clients, and family members. Clinical abilities, experience, and time management
are also important to the manager's success, but without communication skills the manager won't be effective.

Which of the following ECG lead shows changes In IWMI


A. LEAD 2
B.AVL
C.LEAD 1
D. V3

Absence of recognizable QRS Complex in ECG indicate


1)Atrial fibrillation
2)Ventricular fibrillation
3)Sinus arrhythmias
4)paroxysmal VT

Another name Of Glucose?


A. Fruit sugar,
B. Cane Sugar,
C. Fructose,
D. Dextrose,
Answer: D
Daily Requirements of protein Is per Kilogram of Body weight?
A) 1 gm,
B) 10gm,
C) 20gm,
D) 30gm,
Answer: A

A client on prolonged bed rest has developed a pressure ulcer. The wound shows no signs of healing even
though the client has received appropriate skin care and has been turned every 2 hours. Which factor is most
likely responsible for the failure to heal?
A. Inadequate vitamin D intake
B. Inadequate protein intake
C. Inadequate massaging of the affected area
D. Low calcium level
Answer: B
Clients on bed rest suffer from a lack of movement and a negative nitrogen balance.

In Gynace ward , the find out a client, she is on PPH . What the nurse should do first?
A.monitor vitals
B. Call physician
C. Eliminate the blood loss
D. Stay with the client & call for help.
Answer: D
Client may goes to shock. So stay with client
Call help and ask another to call doctor.eliminate blood loss. Then monitor & record vitals

A client complains of abdominal discomfort and nausea while receiving tube feedings. Which intervention is
most appropriate for this problem?
A. Giving the feedings at room temperature
B. Decreasing the rate of feedings and the concentration of the formula
C. Placing the client in semi-Fowler's position while feeding
D. Changing the tube feeding administration set every 24 hours
Answer: B
Decreasing the rate of feedings and the concentration of the formula. Its the higher priority.

which detail of a client's drug therapy is the nurse legally responsible for documenting?
A. Peak concentration time of the drug
B. Safe ranges of the drug
C. Client's socioeconomic data
D. Client's reaction to the drug
Answer: D
The nurse legally must document the client's reaction to the drug in addition to the time the drug was
administered and the dosage given. The nurse isn't legally responsible for documenting the peak concentration
time of the drug, safe drug ranges, or the client's socioeconomic data.

The Ward nurse administering mannitol and the doctor advised slowly to be given. Why?
The risk for ---?
A. cerebral embolism
B. Pulmonary edema
C.hypertension
D. Fluid overload
Answer: B
Rapid fluid shift will results pulmonary edema

The nurse is collecting data on a client before surgery. Which statement by the client would alert the nurse to
the presence of risk factors for postoperative complications?
A "I haven't been able to eat anything solid for the past 2 days."
B. "I've never had surgery before."
C. "I had an operation 2 years ago, and I don't want to have another one."
D. "I've cut my smoking down from two packs to one pack per day."
Answer: D

The physician prescribes morphine 4 mg I.V. every 2 hours as needed for pain. The nurse should be on the alert
for which adverse reaction to morphine?
A. Tachycardia
B. Hypertension
C. Neutropenia
D. Respiratory depression
Answer: D
The nurse should be alert for respiratory depression after morphine administration. Other adverse reactions
include bradycardia (not tachycardia), thrombocytopenia (not neutropenia), and hypotension (not
hypertension).

The nurse is auscultating a client's chest. How can the nurse differentiate a pleural friction rub from other
abnormal breath sounds?
A. A rub occurs during expiration only and produces a light, popping, musical noise.
B. A rub occurs during inspiration only and may be heard anywhere.
C. A rub occurs during both inspiration and expiration and produces a squeaking or grating sound.
D. A rub occurs during inspiration only and clears with coughing.
Answer: C
A male client has been complaining of chest pain and shortness of breath for the past 2 hours. He has a
temperature of 99° F (37.2° C), a pulse of 96 beats/minute, respirations that are irregular and 16
breaths/minute, and a blood pressure of 140/96 mm Hg. He's placed on continuous cardiac monitoring to:
A. prevent cardiac ischemia.
B. assess for potentially dangerous arrhythmias.
C. determine the degree of damage to the heart muscle.
D. evaluate cardiovascular function.
Answer: B

A client with mitral stenosis is scheduled for mitral valve replacement. Which condition may arise as a
complication of mitral stenosis?
A. Left-sided heart failure
B. Myocardial ischemia
C. Pulmonary hypertension
D. Left ventricular hypertrophy
Answer: C

Mitral stenosis, or severe narrowing of the mitral valve, impedes blood flow through the stenotic valve,
increasing pressure in the left atrium and pulmonary circulation.

INR Value of patient with mechanical valves


1)2.5-3.5
2)1-2
3)0-1
4)none of above

A client with chest pain doesn't respond to nitroglycerin (Nitrostat). On admission to the emergency
department, the health care team obtains an electrocardiogram and administers I.V. morphine. The physician
also considers administering alteplase (Activase). This thrombolytic agent must be administered how soon
after onset of myocardial infarction (MI) symptoms?
A. Within 3 to 6 hours
B. Within 24 hours
C. Within 24 to 48 hours
D. Within 5 to 7 days
Answer: A

An unconscious infant received to the emergency department. Which pulse should the nurse palpate during
rapid data collection of an unconscious infant?
A. Radial
B. Brachial
C. Femoral
D. Carotid
Answer: B
The brachial pulse is palpated during rapid data collection of an infant.
During rapid data collection, the nurse's first priority is to check the client's vital functions by checking his
airway, breathing, and circulation.

The nurse places a neonate with hyperbilirubinemia under a phototherapy lamp, covering the eyes and gonads
for protection. The nurse knows that the goal of phototherapy is to:
A. prevent hypothermia.
B. promote respiratory stability.
C. decrease the serum conjugated bilirubin level.
D. decrease the serum unconjugated bilirubin level.
Answer: D
Phototherapy is the primary treatment in neonates with unconjugated hyperbilirubinemia. Photoisomerism is
the therapeutic principle working here.
This conjugated form of bilirubin is then excreted into the bile and removed from the body via the gut/urine.

The physician orders an I.M. injection for a client. Which factor may affect the drug absorption rate from an
I.M. injection site?
A. Muscle tone
B. Muscle strength
C. Blood flow to the injection site
D. Amount of body fat at the injection site
Answer: C
Blood flow to the I.M. injection site affects the drug absorption rate. Muscle tone and strength have no effect
on drug absorption.

What is the most appropriate method to use when drawing blood from a child with hemophilia?
A. Use finger punctures for lab draws.
B. Be prepared to administer platelets for prolonged bleeding.
C. Apply heat to the extremity before venipunctures.
D. Schedule all labs to be drawn at one time.
Answer: D

For a client with cirrhosis, deterioration of hepatic function is best indicated by:
A. fatigue and muscle weakness.
B. difficulty in arousal.
C. nausea and anorexia.
D. weight gain.
Answer: B
Hepatic encephalopathy, a major complication of advanced cirrhosis, occurs when the liver no longer can
convert ammonia (a by-product of protein breakdown) into glutamine. This leads to an increased blood level of
ammonia — a central nervous system toxin — which causes a decrease in the level of consciousness. Fatigue,
muscle weakness, nausea, anorexia, and weight gain occur during the early stages of cirrhosis.
1.Cellulitis on the floor of mouth is known as...???
A. Stomatitis
B. Glositis
C. Angina pectoris
D. Angina Ludovici
E. Gingivitis
Answer:D/ ludwing's angina

2. To remove soft contact lenses from the eyes of an unconscious patient the nurse should:
A. Uses a small suction cup placed on the lenses
B. Pinches the lens off the eye then slides it off the cornea
C. Lifts the lenses with a dry cotton ball that adheres to the lenses
D. Tenses the lateral canthus while stimulating a blink reflex by the patient
Answer:B

3.A patient undergoes laminectomy. In the immediate post-operative period, the nurse should
A. Monitor the patient's vital signs and log roll him to prone position
B. Monitor the patient's vital signs and encourage him to ambulate
C. Monitor the patient's vital signs and auscultate his bowel sounds
D. Monitor the patient's vital signs, check sensation and motor power of the feet
Answer:D

4. A patient with duodenal peptic ulcer would describe his pain as:
A. Generalized burning sensation
B. Intermittent colicky pain
C. Gnawing sensation relieved by food
D. Colicky pain intensified by food
Answer:D

5. A patient admitted to the hospital in hypertensive crisis is ordered to receive hydralazine


(Apresoline) 20mg IV stat for blood pressure greater than 190/100 mmHg. The best response of the
Nurse to this order is to:
A. Give the dose immediately and once
B. Give medication if patient's blood pressure is > 190/100 mmHg
C. Call the physician because the order is not clear
D. Administer the dose and repeat as necessary
Answer:A

6. Whilst recovering from surgery a patient develops deep vein thrombosis. The sign that would indicate this
complication to the nurse would be:
A. Intermittent claudication
B. Pitting edema of the area
C. Severe pain when raising the legs
D. Localized warmth and tenderness of the site
Answer:D

7. A patient presents to the emergency department with diminished and thready pulses,hypotension and an
increased pulse rate. The patient reports weight loss, lethargy, and decreased urine output. The lab work
reveals increased urine specific gravity. The nurse should suspect:
A. Renal failure
B. Sepsis
C. Pneumonia
D. Dehydration
Answer:D

8.client with chronic renal failure has a serum potassium level of 6.8 mEq/L. What should the nurse assess
first?
A. Blood pressure
B. Respirations
C. Temperature
D. Cardiac rhythm
Answer: D

9.The nurse is planning care for a client with pneumococcal pneumonia. Which of the following would be most
effective in removing respiratory secretions?
A. Administration of cough suppressants
B. Increasing oral fluid intake to 3000 cc per day
C. Maintaining bed rest with bathroom privileges
D. Performing chest physiotherapy twice a day
Answer is B: Increasing oral fluid intake to 3000 cc per day. Secretion removal is enhanced with adequate
hydration which thins and liquefies secretions.

10.Method to diagnosis & locate seizures?


A. EEG
B. PET
C. MRI
D. CT scan
Answer: A
11.The primary goal of therapy for a client with pulmonary edema and heart failure?
A Enhance comfort
B Improve respiratory status
C Peripheral edema decreased
D Increase cardiac output
Answer: D

12.The nurse is preparing to administer an I.M. injection in a client with a spinal cord injury that has resulted in
paraplegia. Which of the following muscles is best site for the injection in this case?
A. Deltoid.
B. Dorsal gluteal.
C. Vastus lateralis.
D. Ventral gluteal.
Answer: A

13. The nurse is to collect a sputum specimen from a client. The best time to collect this specimen is:
A. early in the evening.
B. anytime during the day.
C. in the morning, as soon as the client awakens.
D. before bedtime.
Answer: C Because sputum accumulates in the lungs during sleep, the nurse should collect a sputum specimen
in the morning, as soon as the client awakens and before he eats or drinks. This specimen will be concentrated,
increasing the likelihood of an accurate culture

14. An obese client has returned to the unit after receiving sedation and electroconvulsive therapy. The nurse
requests assistance moving the client from the stretcher to the bed. There are 2 people available to assist.
Which of the following is the best method of transfer for this patient?
A. Carry lift.
B. Sliding board.
C. Lift sheet transfer.
D. Hydraulic lift.
Answer:B

Which type of nursing intervention does the nurse perform when she administers oral care to a client?
A. Psychomotor.
B. Educational.
C. Maintenance.
D. Supervisory.
Answer:c
On her 3rd postpartum day, a client complains of chills and aches. Her chart shows that she has had a
temperature of 100.6° F (38.1° C) for the past 2 days. The nurse assesses foul-smelling, yellow lochia. What do
these findings suggest?
A. Lochia alba
B. Lochia serosa
C. Localized infection
D. Cervical laceration

What is the term used for normal respiratory rhythm and depth in a client?
A. Eupnea
B. Apnea
C. Bradypnea
D. Tachypnea

QJ1. A client receives a painkiller. Thirty minutes


later , The nurse asks the client if the pain is
relieved. Which step of nursing process the
nurse is using?
A. Assessment
B. Nursing diagnosis
C. Implementation
D. Evaluation

A client says to the nurse "I know that I'm going to die." Which of the following responses by the nurse would
be best?
A. "We have special equipment to monitor you and your problem."
B. "Don't worry. We know what we're doing and you aren't going to die."
C. "Why do you think you're going to die?"
D. "Oh no, you're doing quite well considering your condition."

A dull percussion is noted over the symphysis pubis , it may indicate


A. Pelvic inflammatory disease
B. Prostatitis
C. Peritonitis
D. Distended Bladder
Answer: D

The nurse is assessing the reflexes of a newborn. The nurse assesses which of the following reflexes by placing
a finger in the newborn’s mouth?
A. Moro reflex
B. Sucking reflex
C. Rooting reflex
D. Babinski reflex
Answer: B

When caring for a patient who has intermittent claudication, a cardiac/vascular nurse advises the patient to:
A. apply graduated compression stockings before getting out of bed.
B. elevate the legs when sitting.
C. refrain from exercise.
D. walk as tolerated.
Answer: D

The client is brought to the emergency department due to drug poisoning. Which of the following nursing
interventions is most effective in the management of the client’s condition?
a) Gastric lavage
b) Activated charcoal
c) Cathartic administration
d) Milk dilution
Answer:B Activated charcoal
The administration of activated charcoal is the most effective in the management of poisoning because it
absorbs chemicals in the gastrointestinal tract, thus reducing its toxicity.

A nurse is assessing a group of clients. The nurse knows that which of the following clients is at risk for fluid
volume deficit?(DHA)
a) Client diagnosed with liver cirrhosis.
b) Client with diminished kidney function.
c) Client diagnosed with congestive heart failure.
d) Client attached to a colostomy bag.
Answer: D

Best time to check IOP?


A. Early morning
B. After noon
C. Late evening
D. At noon
Answer: A

The physician teaches a client about the need to increase her intake of calcium. At a follow-up appointment,
the nurse asks the client which foods she has been consuming to increase her calcium intake. Which answer
suggests that teaching about calcium-rich foods was effective?
A. Broccoli and nuts
B. Yogurt and kale
C. Bread and shrimp
D. Beans and potatoes
Answer: B

The nurse is caring for a client diagnosed with a stroke. Because of the stroke, the client has dysphagia
(difficulty swallowing). Which intervention by the nurse is best for preventing aspiration?
A. Placing the client in high Fowler's position to eat.
B. Offering liquids and solids together.
C. Keeping liquids thinned.
D. Placing food on the affected side of the mouth.
Answer: A

When administering an I.M. injection to an infant, the nurse in charge should use which site?
a. Deltoid
b. Dorsogluteal
c. Ventrogluteal
d. Vastus lateralis
Answer: D

Which organ in the body always recieve the most percentage of blood(%cardiac output)flow?.
(AIIMS,ME,BPSC )
A. Kidney
B. Heart
C. Brain
D. Lung
Answer: D
Lung recieves 100% of cardiac output via both pulmonary & systemic circulation.

The hormone responsible for a positive pregnancy test (UPT)is:


A. Estrogen
B. Progesterone
C. Human Chorionic Gonadotropin
D. Follicle Stimulating hormone
Answer: C

The stool guaiac test (gFOBT) for the detection of...?


A. Piles
B. Peptic ulcer
C. Intestinal obstruction
D. Colo-Rectol carcinoma
Answer: D

Which of the following, if observed as a sudden change in the resident, is considered a possible warning sign of
a stroke?
A. Dementia
B. Contractures
C. Slurred speech
D. Irregular heartbeat
Answer:C
One of the clasical symptom of stroke

A resident who is incontinent of urine has an increased risk of developing (prometric saudi2016)
A. dementia.
B. urinary tract infections.
C. dehydration
D. pressure sore
Answer
Risk for altered skin integrity due to contact with wet surface

A resident is on a bladder retraining program. The nurse aide can expect the resident to
A . Have a fluid intake restriction to prevent sudden urges to urinate.
B . Wear an incontinent brief in case of an accident.
C . Have an indwelling urinary catheter.
D . Have aschedule for toileting.

What is the first choice of MI


a)Ecospirin
b)Streptokinase
c)Morphine
c)Heparin

In a client with chronic bronchitis, which sign would lead the nurse to suspect right-sided heart failure?
A. Cyanosis of the lips
B. Bilateral crackles
C. Productive cough
D. Leg edema

Which of the following is considered a normal age-related change?


A. Dementia
B. Contractures
C. Bladder holding less urine
D. Wheezing when breathing
Answer: C
Is an age related physiological changes. Othes are pathological
Which is the primary consideration when preparing to administer thrombolytic therapy to a patient who is
experiencing an acute myocardial infarction (MI)?(HAAD2014)
A. History of heart disease.
B. Sensitivity to aspirin.
C. Size and location of the MI.
D. Time since onset of symptoms.
Answer: D
Its the crieteria for thrombolytic therapy, early onset.Thrombolytic medications are approved for the
immediate treatment of stroke (with in 3hrs of onset)and heart attack(with in 12 hrs of onest)

Which of the following statements is true about range of motion (ROM) exercises?
A Done just once a day
B Help prevent strokes and paralysis
C Require at least ten repetitions of each exercise
D Are often performed during ADLs such as bathingor dressing
Answer:D

When instructing the client diagnosed with hyperparathyroidism about diet, the nurse should stress the
importance of:
A. restricting fluids.
B. restricting sodium.
C. forcing fluids.
D. restricting potassium.

. When assessing a client with glaucoma, a nurse expects which of the following findings?
A. Complaints of double vision.
B. Complaints of halos around lights.
C. Intraocular pressure of 15 mm Hg.
D. Soft globe on palpation.

In the emergency department, the nurse is caring for a client with type 1 diabetes who was brought in by
ambulance after losing consciousness. Upon assessment, the client's breath was noted to be fruity. Which of
the following ABG results would the nurse expect?
A. pH: 7.49 PCO2: 50 HCO3: 18
B. pH 7.28: PCO2: 40 HCO3: 16
C. pH:7.38 PCO2: 45 HCO3: 26
D. pH: 7.31 PCO2: 60 HCO3: 29
Answer:B
Risk for metabolic acidosis in type1 DM

Before administering methergine , to treat PPH the nursing priority to check


A. Uterine tone
B. Output
C. BP
D. amount of lochia
E. Deep tendon reflex
Answer: C

High risk clients for the reactivation of herpes zoster?


the clients with ....
A. First degree burns
B. Renal transplant
C. Post ORIF
D. Head injury.

The cardiac marker which is elevated soon after MI is


A:Trop-T
B:CKMB
C:LDH
D:Myoglobin

The nurse is taking the health history of a patient being treated for sickle cell disease. After being told the
patient has severe generalized pain, the nurse expects to note which assessment finding?
A. Severe and persistent diarrhea
B. Intense pain in the toe
C. Yellow-tinged sclera
D. Headache

A client has hypoxemia of pulmonary origin. What portion of arterial blood gas results is most useful in
distinguishing acute respiratory distress syndrome from acute respiratory failure?
A. Partial pressure of arterial oxygen (PaO2)
B. Partial pressure of arterial carbon dioxide (PaCO2)
C. pH
D. Bicarbonate (HCO3–)
Answer: A

The procedure involves removal of the "head" (wide part) of the pancreas, the duodenum, a portion of the
common bile duct, gallbladder, and sometimes part of the stomach.And anastomosis to jejunum ?
A. Birloth 1procedures
B. Birloth 2 procedures
C. Wipple procedures
D. Subtotal cholecystectomy
Answer: C
A client with a fluid volume deficit is receiving an I.V. infusion of dextrose 5% in water and lactated Ringer's
solution at 125 ml/hour. Which data collection finding indicates the need for additional I.V. fluids?
A. Serum sodium level of 135 mEq/L
B. Temperature of 99.6° F (37.6° C)
C. Neck vein distention
D. Dark amber urine
Answer: D
Normally, urine appears light yellow; dark amber urine is concentrated and suggests decreased fluid intake.

Pseudo membraneous colitis is due to


A. Tetenus toxins
B. Clostridium difficile
C. H.pylori
D. E- Coli

Glomerulonephritis is the complication of impetigo due to...?


A. Streptococcus
B. Staphylococci
C. Pseudomonas
D. Klebsiella

Which of the following types of immunoglobulins does not cross the barrier between mother and infant in the
womb?
A. IgA
B. IgM
C. IgD
D. IgE

The most severe expressions of alcohol withdrawal syndrome?


A. disequilibrium syndrome
B. dawn phenomenon
C. somogyi phenomenon
D. Delirium tremens
Answer : D

A 39-year-old forklift operator presents with shakiness, sweating, anxiety, and palpitations and tells the nurse
he has type 1 diabetes mellitus. Which of the follow actions should the nurse do first?
A. Inject 1 mg of glucagon subcutaneously.
B. Administer 50 mL of 50% glucose I.V.
C. Give 4 to 6 oz (118 to 177 mL) of orange juice.
D. Give the client four to six glucose tablets

The nurse is collecting data on a male client diagnosed with gonorrhea. Which symptom likely prompted the
client to seek medical attention?
A. Rashes on the palms of the hands and soles of the feet
B. Cauliflower-like warts on the penis
C. Painful red papules on the shaft of the penis
D. Foul-smelling discharge from the penis

A client with B negative blood requires a blood transfusion during surgery. If no B negative blood is available,
the client should be transfused with:
A. A positive blood
B. B positive blood
C. O negative blood
D. AB negative blood
Answer: C
If the client’s own blood type and Rh are not available, the safest transfusion is O negative blood. Answers A, B,
and D are incorrect because they can cause reactions that can prove fatal to the client

An woman is prescribed metformin for glucose control. The patient is on NPO status pending a diagnostic test.
The nurse is most concerned about which side effect of metformin?
A. Diarrhea and Vomiting
B. Dizziness and Drowsiness
C. Metallic taste
D. Hypoglycemia

A 30-year-old male from Haiti is brought to the emergency department in sickle cell crisis. What is the best
position for this client?
a. Side-lying with knees flexed
b. Knee-chest
c. High Fowler’s with knees flexed
d. Semi-Fowler’s with legs extended on the bed

Which of the following vein is commonly used for CABG


a)Femoral
b)greater saphenus
c)popliteal
d)brachial

Chronic alcoholic's needs which of the following vitamins


A. Thiamine
B. Riboflavine
C .cyanocobalamin
D. Pyridoxine
Answer : A

which of the following is the warning sign of dying?


A. Rigor mortis
B. Kussmaul breathing
C. Chyene stroke respiraton
D. Tachycardia
Answer: C

The nurse administers furosemide (Lasix) to treat a client with heart failure. Which adverse effect must the
nurse watch for most carefully?
A. Increase in blood pressure
B. Increase in blood volume
C. Low serum potassium level
D. High serum sodium level
Answer: C
Furosemide is a potassium-wasting diuretic. The nurse must monitor the serum potassium level and assess for
signs of low potassium. As water and sodium are lost in the urine, blood pressure decreases, blood volume
decreases, and urine output increases.

A confirmational test for gestational diabetes ?


A. Fasting blood sugar
B. Urine sugar
C.glucose tolerance test
D. Fasting lipid profile with RBS
Answer: C

nasal septum disruption is an indication for over usage of ---


A. Marijuana
B. Alcohol
C. cocaine
D. Brown sugar
Answer: C

The nurse is caring for a client with pneumonia. The physician orders 600 mg of ceftriaxone (Rocephin) oral
suspension to be given once per day. The medication label indicates that the strength is 150 mg/5ml. How
many milliliters of medication should the nurse pour to administer the correct dose?
A. 2.5 ml
B. 4 ml
C. 10 ml
D. 20 ml
Answer: D

The nurse is preparing to discharge a 70-year-old man on warfarin therapy for a pulmonary embolism. The
nurse’s dischargeteaching should include which of the following instructions?
A.Follow a healthy diet by increasing ingestion of green, leafy vegetables.
B. Take herbal remedies to manage cold symptoms.
C. Avoid alcohol due to enhanced anticoagulant effect.
D. Take Coumadin only on an empty stomach.

Route of administration of BCG vaccine?question (AIIMS Delhi 2011)


A. IM
B. ID
C. SC
D. IV
Answer: B

major part of cardiac output used for which organ?


A. Brain
B. Heart
C. Spleen
D.kidney

Cystic fibrosis is diagnosed by ....


A. Fibro scan
B. Sweat test
C. Myelogram
D. USG

World immunization day?


A. Oct 10
B. Nov 10
C. Dec 10
D. Aug 10
Answer: B

A client with a myocardial infarction and cardiogenic shock is placed on an intra-aortic balloon pump (IAPB). If
the device is functioning properly, the balloon inflates when the:
A. tricuspid valve is closed.
B. pulmonic valve is open.
C. aortic valve is closed.
D. mitral valve is closed

QM1: world malaria day?


A. March 18
B. April 25
C. May 12
D. May 25

World Cancer day is ...?


A. January 30
B. December 1
C. February 4
D. March 7
Answer: C

In adults, Normal endotracheal suction pressure?


A. 40-90 mm of Hg
B. 70-140 mm of Hg
C. 130- 180 mm of Hg
D. 180-320 mm of Hg
Answer: B

A client undergoes hip-pinning surgery(DHS) to treat an intertrochanteric fracture of the right hip. The nurse
should include which intervention in the postoperative plan of care?
A. Performing passive range-of-motion (ROM) exercises on the client's legs once each shift
B. Keeping a pillow between the client's legs at all times
C. Turning the client from side to side every 2 hours
D. Maintaining the client in semi-Fowler's position
Answer: B

During the initial admission process, a geriatric client seems confused. What is the most probable cause of this
client's confusion?
A. Depression
B. Altered long-term memory
C. Decreased level of consciousness (LOC)
D. Stress related to an unfamiliar situation
Answer: D
The stress of being in an unfamiliar situation, such as admission to a hospital, can cause confusion in geriatric
clients. Depression doesn't produce confusion, but it can cause mood changes, weight loss, anorexia,
constipation, and early morning awakening. In geriatric clients, long-term memory usually remains intact,
although short-term memory may be altered. Decreased LOC doesn't normally result from aging; therefore, it's
a less likely cause of confusion in this client.

The physician orders an I.M. injection for a client. Which factor may affect the drug absorption rate from an
I.M. injection site?
A. Muscle tone
B. Muscle strength
C. Blood flow to the injection site
D. Amount of body fat at the injection site
Answer: C
Blood flow to the I.M. injection site affects the drug absorption rate. Muscle tone and strength have no effect
on drug absorption. The amount of body fat at the injection site may help determine the size of the needle and
the technique used to localize the site; however, it doesn't affect drug absorption (unless the nurse
inadvertently injects the medication into the subcutaneous tissue instead of the muscle).

The nursing care plan for a client with decreased adrenal function should include
A. Encouraging activity
B. Placing client in reverse isolation
C. Limiting visitors
D. Measures to prevent constipation
Answer is C: Limiting visitors
Any exertion, either physical or emotional, places additional stress on the adrenal glands which could
precipitate an addisonian crisis. The plan of care should protect this client from the physical and emotional
exertion of visitors.

Witch's milk commonly seen only in ----?


A.Infants
B.newborns
C. Adolescents
D. prenatal clients
Answer: A

Lactation provides contraception for


A. 14 days
B. 30 days
C. 90 days
D. 120 days
E. 180 days
Answer: C
The nurse is doing a physical assessment and electrocardiogram on an elderly client. Which finding during the
nurse's assessment of the cardiac system is of most concern and warrants prompt further investigation?
A. S4 heart sound.
B. Increased PR interval.
C. Orthostatic hypotension.
D. Irregularly irregular heart rate.

An elderly patient is admitted to the hospital with swollen ankles. The best way to limit
edema of the feet is for the nurse to:

A. Restrict fluids
B. Apply bandages
C. Elevate the legs
D. Do passive range of motion exercises (ROM)
Answer : C

Which of the following actions is the most effective measure to reduce hospital acquired
infections?

A. Double bagging of all contaminated laundry


B. Restricting visitors of infectious patients
C. Using disposable supplies
D. Correct hand washing
Answer : D

A patient is diagnosed with diabetic ketoacidosis. The nurse would expect the physician to
prescribe:

A. Regular insulin IV
B. NPH insulin SC
C. Glucagon IM
D. Mixed insulin SC
Answer : A

The rationale for having the patient void before an abdominal paracentesis procedure is to:

A. Minimize discomfort
B. Avoid abdominal distention
C. Prevent bladder puncture
D. Reduce infection rate
Answer : C
An 85-year-old man is admitted with dementia. He continuously attempts to remove his
nasogastric tube. The nurse applies cloth wrist restraints as ordered. Which of the following
actions by the nurse is most appropriate?

A. Evaluate the need to restrain by observing patient’s behavior once every 24 hrs
B. Perform circulation checks to the extremities every two hours
C. Remove the restraints when the patient is sleeping
D. Instruct family to limit physical contact with the patient
Answer : B

During balloon inflation of an indwelling urinary catheter, the patient complains of pain and
discomfort. The nurse should:

A. Continue the procedure and assure the patient


B. Aspirate the fluid and remove the catheter
C. Withdraw the fluid and reinsert the catheter
D. Decrease the amount of injected fluid and secure
Answer : C

A patient is to receive 25mg/hr of an aminophylline infusion. The solution prepared by the


pharmacy contains 500mg of aminophylline in 1000ml of D5W. How many milligrams are
available per ml?

A. 0.25 mg/ml
B. 0.5 mg/ml
C. 1 mg/ml
D. 2 mg/ml
Answer : B

A patient has had a total hip joint replacement. Which of the following actions should the
nurse consider for the patient’s daily recommended exercise program?

A. Administering an analgesic before exercising


B. Discontinuing the program if the patient dislikes it
C. Continuing exercises inspire of severe pain
D. Evaluating effectiveness of exercise based on pain scale
Answer : A
Whilst recovering from surgery a patient develops deep vein thrombosis. The sign that
would indicate this complication to the nurse would be:

A. Intermittent claudication
B. Pitting edema of the area
C. Severe pain when raising the legs
D. Localized warmth and tenderness of the site
Answer : D

Which of the following techniques should the nurse implement to prevent the patient’s
mucous membranes from drying when the oxygen flow rate is higher than 4 liters per
minute?

A. Use a non rebreather mask


B. Add humidity to the delivery system
C. Use a high flow oxygen delivery system
D. Ensure that the prongs are in the nares correctly
Answer : B

Extrapyramidal adverse effects and symptoms are most often associated with which of the
following drug classes?

A. Antidepressants
B. Antipsychotics
C. Antihypertensives
D. Antidysrhythmics
Answer : B

A construction worker was brought to the emergency department and admitted with the
diagnosis of heat stroke due to strenuous physical activity during hot weather conditions.
Which action should the nurse take?

A. Immediately immerse the patient in cold water to reduce the patient’s temperature
B. Administer an antipyretic such as aspirin or acetaminophen
C. Place ice packs to the neck, axillae, scalp and groin
D. Encourage foods and oral fluids that contain carbohydrates and electrolytesAnswer : C
The nurse should observe for which of the following symptoms in a patient who has just
undergone a total thyroidectomy:

A. Weight gain
B. Depressed reflexes
C. Muscle spasm and twitching
D. Irritable behavior
Answer : C

Which of the following indicates the nurse is engaging in a therapeutic nurse-patient


relationship?

A. The nurse establishes a relationship that is mutually beneficial


B. The nurse demonstrates sympathetic feelings toward the patient
C. The nurse commits to helping the patient find ways to help self
D. The nurse utilizes therapeutic touch to convey acceptance of the patient
Answer : C

One factor affecting the pharmokinetics of older patients’ drug absorption is:

A. Decreased gastrointestinal motility


B. A difficulty in swallowing
C. A prevalence of obesity
D. Numerous medications
Answer : A

Question No : 36 –

Pain management for terminally ill patients is most effective when analgesics are given:

A. Around the clock


B. Only when clearly needed
C. After non-pharmacological methods fail
D. As the patient requests them
Answer : A

Question No : 37 –
The physician orders 20 u of U-100 regular insulin. The only syringe on hand is a 1 ml
tuberculin syringe. How many milliliters should be administered?

A. 0.02 ml
B. 0.2 ml
C. 1 ml
D. 2 ml
Answer : B

Question No : 38 –

The urinary catheter is kept securely in the bladder by:

A. Taping the urinary catheter to the leg


B. Securing catheter and collection bag connections
C. Inflating the balloon of the catheter
D. Anchoring the catheter bag to the bed
Answer : C

Question No : 39 –

Order: Compazine 8 mg IM stat. Drug availablE.10 mg/ 2mL in vial.


How many mLs would you give?

A. 0.6 mL
B. 1.6 mL
C. 2.6 mL
D. 3.6 mL
Answer : B

Question No : 40 –

A medication was ordered by a physician. The nurse believes the medication dose is
incorrect. What should the nurse do next?

A. Clarify the order with another physician who is available on the unit
B. Ask the nurse in charge if the order is correct
C. Contact the pharmacy department
D. Call the physician who prescribed the medication
Answer : D

Question No : 41 –

The immediate treatment for ventricular fibrillation is:

A. Precordial blow
B. Defibrillation
C. Bolus of lidocaine
D. Ventricular pacing
Answer : B

Question No : 42 –

A patient requires tracheal suctioning through the nose. Which of the following nursing
action would be incorrect?

A. Lubricating the catheter with sterile water


B. Applying suction while withdrawing the catheter from the nose
C. Applying suction for a minimum of 30 seconds
D. Rotating the catheter while withdrawing it
Answer : C

Question No : 43 –

Thirty minutes after starting a blood transfusion a patient develops tachycardia and
tachypnea and complains of chills and low back pain. The nurse recognizes these
symptoms as characteristic of:

A. Circulatory overload
B. Mild allergy
C. Febrile response
D. Hemolytic reaction
Answer : D
Question No : 44 –

To remove soft contact lenses from the eyes of an unconscious patient the nurse should:

A. Uses a small suction cup placed on the lenses


B. Pinches the lens off the eye then slides it off the cornea
C. Lifts the lenses with a dry cotton ball that adheres to the lenses
D. Tenses the lateral canthus while stimulating a blink reflex by the patient
Answer : B

Question No : 45 –

Order: 1000 ml of D5W to run for 12 hours. Using a micro drip set calculate the drops per
minute (gtts/min):

A. 20 gtts/min
B. 45 gtts/min
C. 60 gtts/min
D. 83 gtts/min
Answer : D

Question No : 46 –

Which of the following tasks requires specialized education and should be performed by
the nurse only after the training has been completed?

A. Administering a dose of promethazine (Phenergan) via intravenous push (IVP)


B. Applying a transdermal fentanyl (Duragesic)
C. Instilling tobramycin (Tobrex) ophthalmic solution
D. Beginning an intravenous infusion of cyclophosphamide (Cytoxan)
Answer : D

Question No : 47 –

The patient is to receive 100 ml/hr of D5W through a micro drip. How many drops per
minute should the patient receive?

A. 25 gtts/min
B. 30 gtts/min
C. 100 gtts/min
D. 200 gtts/min
Answer : C

Question No : 48 –

Immediately after a craniotomy for head trauma, the nurse must monitor the drainage on
the dressing. Which of the following should be reported?

A. Blood tinged
B. Straw colored
C. Clotted
D. Foul-smelling
Answer : B

Question No : 49 –

Which of the following interventions should the nurse implement if a patient complains of
cramps while irrigating the colostomy?

A. Reduce the flow of solution


B. Have the patient sit up in bed
C. Remove the irrigation tube
D. Insert the tube further into the colon
Answer : A

Question No : 50 –

A nurse is not familiar with a particular solution ordered to irrigate a patient’s wound. The
appropriate action would be to:

A. Check if the solution is available on the ward, and if so, use it to clean the wound
B. Put a neat line through the order and re-write the solution more commonly used
C. Check with the Pharmacist about the uses of the solution ordered
D. Ask the patient what solution he would prefer to be used
Answer : C

Question No : 51 –

A 65-year-old patient is admitted with ischemic stroke. Which of the following would be
initially assessed by the nurse to determine the patients level of consciousness?

A. Visual fields
B. Deep tendon reflexes
C. Auditory acuity
D. Verbal response
Answer : D

Question No : 52 –

While preparing for a kidney biopsy the nurse should position the patient:

A. Prone with a sandbag under the abdomen


B. Lateral opposite to biopsy site
C. Supine in bed with knee flexion
D. Lateral flexed knee-chest
Answer : A

Question No : 53 –

To promote accuracy of self-monitoring blood glucose by patients the nurse should:

A. Retrain patients periodically


B. Direct patients to rotate testing sites
C. Advise patients to buy new strips routinely
D. Compare results from patient’s meter against lab results
Answer : A
Question No : 54 –

After administration of penicillin, a patient develops respiratory distress and severe


bronchospasm. The nurse should:

A. Contact the physician


B. Apply ice packs to the axilla
C. Assess the patient for orthostatic hypotension
D. Encourage the patient to take slow deep breaths
Answer : A

Question No : 55 –

The administration of which of the following types of parenteral fluids would result in a
lowering of the osmotic pressure and cause the fluid to move into the cells?

A. Hypotonic
B. Isotonic
C. Hypertonic
D. Colloid
Answer : A

Question No : 56 –

A newborn infant is assessed using the Apgar assessment tool and scores 6. The infant
has a heart rate of 95, slow and irregular respiratory effort, and some flexion of extremities.
The infant is pink, but has a weak cry. The nurse should know that this Apgar score along
with the additional symptoms indicates the neonate is:

A. Functioning normally
B. Needing immediate life-sustaining measures
C. Needing special assistance
D. Needing to be warmed
Answer : C

Question No : 57 –

Nursing management of the patient with external otitis includes:


A. Irrigating the ear canal with warm saline several hours after instilling lubricating ear drops
B. Inserting an ear wick into the external canal before instilling the ear drops to disperse the medication
C. Teaching the patient how to instill antibiotic drops into the ear canal before swimming
D. Instilling ear drops without the dropper touching the auricle and positioning the ear upward for 2 minutes
afterwards
Answer : D

Question No : 58 –

The best example of documentation of patient teaching regarding wound care is:

A. “The patient was instructed about care of wound and dressing changes”
B. “The patient demonstrated correct technique of wound care following instruction”
C. “The patient and family verbalize that they understand the purposes of wound care”
D. “Written instructions regarding wound care and dressing changes were given to the patient”
Answer : B

Question No : 59 –

A patient is ordered 75mg of pethidine which comes in an ampoule of 100mg/2ml. What


would the nurse do with the remaining pethidine after drawing up the required dose?

A. Lock up the remaining dose in the medication cupboard to use later


B. Ask a fellow staff nurse to witness the disposal of the remaining drug
C. Use the remaining dose within 2 hours for another patient
D. Pour the remaining dose down the nearest sink
Answer : B

Question No : 60 –

A nurse can ensure she maintains her competency to practice through:

A. Being involved in continuing education programs


B. Making sure that what was learnt at nursing school is strictly followed
C. Closely carrying out instructions given by the Charge Nurse
D. Working on the same ward for at least 2 years
Answer : A
Question No : 61 –

The patient with liver cirrhosis receives 100 ml of 25% serum albumin intravenously. Which
of the following findings would best indicate that the albumin is having its desired effect?

A. Decreased blood pressure


B. Increased serum albumin level
C. Increased urine output
D. Improved breathing pattern
Answer : C

Question No : 62 –

The nurse should suspect that a patient has bleeding in the upper gastrointestinal tract
when the color of the patient’s stool is:

A. Yellow
B. Black
C. Clay
D. Red
Answer : B

Question No : 63 –

A registered nurse delegates care to a practical nurse. The person most responsible for the
safe performance of the care is the:

A. Head nurse who is in-charge of the unit


B. The practical nurse assigned to provide the care
C. The registered nurse who delegated the care to the practical nurse
D. The nursing care coordinator who is the supervisor of the unit
Answer : C

Question No : 64 –

A deficiency of which of the following vitamins can affect the absorption of calcium?
A. Vitamin C
B. Vitamin B6
C. Vitamin D
D. Vitamin B12
Answer : C

Question No : 65 –

A patient with a central venous line develops sudden clinical manifestations that include a
decrease in blood pressure, an elevated heart rate, cyanosis, tachypnea, and changes in
mental status. Which of the following is the most likely cause of these symptoms?

A. An air embolism
B. Circulatory overload
C. Venous thrombosis
D. Developing bacteremia
Answer : A

Question No : 66 –

When taking routine post-operative observations on a patient who underwent an


exploratory laparotomy, the nurse plans to monitor which important finding over the next
hour?

A. Serosanguinous drainage on the surgical dressing


B. Blood pressure of 105/65 mmHg
C. Urinary output of 20 mls in the last hour
D. Temperature of 37.6 °C
Answer : C

Question No : 67 –

When the nurse is caring for a patient placed on droplet precautions, the nurse should:

A. Have the patient wear a high-efficiency particulate air (HEPA) mask


B. Wear a surgical mask when standing within 3 feet (1 meter) of the patient
C. Assign the patient to a room with monitored negative air pressure
D. Apply a disposable gown when entering the patient’s room
Answer : B

Question No : 68 –

A patient who has just had a miscarriage at 8 weeks of gestation is admitted to hospital. In
caring for this patient, the nurse should be alert for signs of:

A. Dehydration
B. Subinvolution
C. Hemorrhage
D. Hypertension
Answer : C

Question No : 69 –

Collection urine bag should be emptied as necessary and at least every 8 – 9 hours to
prevent:

A. Pooling of urine in the tube


B. Reflux of urine into the bladder
C. Pulling on catheter
D. Bacterial contamination
Answer : D

Question No : 70 –

Which of the following statements accurately describes the occurrence of dyspnea in


patients who are receiving end of life care?

A. Dyspnea is only experienced by patients who have primary diagnoses that involve the lungs
B. Dyspnea occurs in less than 50% of the patients who are receiving end of life care
C. Dyspnea that is caused by increased fluid volume may be improved by diuretics
D. Dyspnea may be caused by antibiotic therapy used over a long period of time
Answer : C

Question No : 71 –
The patient has a nursing diagnosis of altered cerebral tissue perfusion related to cerebral
edema. An appropriate nursing intervention for this problem is to:

A. Elevate the head of the bed 30 degrees


B. Provide a position of comfort with knee flexion
C. Provide uninterrupted periods of rest
D. Ensure adequate hydration with mannitol
Answer : A

Question No : 72 –

While assessing a patient, the nurse learns that he has a history of allergic rhinitis, asthma,
and multiple food allergies. The nurse must:

A. Be alert to hypersensitivity response to the prescribed medications


B. Encourages the patient to carry an epinephrine kit in case of an allergic reaction
C. Advise the patient to use aspirin in case of febrile illnesses
D. Admit the patient to a single room with limited exposure to health care personnel
Answer : A

Question No : 73 –

The nurse should administer nasogastric tube (NGT) feeding slowly to reduce the hazard
of:

A. Distention
B. Abdominal cramps
C. Diarrhea
D. Regurgitation
Answer : A

Question No : 74 –

A patient arrived to the Post Anesthesia Care Unit (PACU) complaining of pain after
undergoing a right total hip arthroplasty. Which of the following should the nurse do to
assess the patient’s level of pain?
A. Determine the patient’s position during surgery and how long the patient was in this position
B. Inspect the dressing, note type and amount of drainage, and insure bandage adhesive is not pulling on skin
C. Ask anesthesiologist what type of anesthesia patient received and last dose of pain medication
D. Note location, intensity and duration of pain and last dose and time of pain medication
Answer : D

Question No : 75 –

When caring for a patient with impaired mobility that occurred as a result of a stroke (right
sided arm and leg weakness). The nurse would suggest that the patient use which of the
following assistive devices that would provide the best stability for ambulating?

A. Crutches
B. Single straight-legged cane
C. Quad cane
D. Walker
Answer : C

Question No : 76 –

The nurse teaches a patient recovering from a total hip replacement that it is important to
avoid:

A. Putting a pillow between the legs while sleeping


B. Sitting with the legs crossed
C. Abduction exercises of the affected leg
D. Bearing weight exercises on the affected leg for 6 weeks
Answer : B

Question No : 77 –

A patient with duodenal peptic ulcer would describe his pain as:

A. Generalized burning sensation


B. Intermittent colicky pain
C. Gnawing sensation relieved by food
D. Colicky pain intensified by food
Answer : C

Question No : 78 –

You have started work on a new ward. One of the patient’s allocated to you has been on
the ward for the last 7 months since she had a cerebrovascular accident (CVA). You notice
that her nursing care plan says strict bed rest, but on assessment you can not see any
reason why this patient can not sit out of bed for short periods. Your nursing action would
be:

A. Check with the other nursing staff as to reasons behind the nursing care plan then update the plan based on
your assessment
B. Follow the nursing care plan strictly as this would have been developed after a detailed and collaborative
assessment
C. Seek physician’s orders so that you have permission to move the patient
D. Try and move the patient without consulting with anyone to see how she manages
Answer : A

Question No : 79 –

A nurse prepares a narcotic analgesic for administration, but the patient refuses to take it.
Which of the following actions by the nurse is most appropriate?

A. Encourage the patient to reconsider taking the medication


B. Label the medication and replace it for use at a later time
C. Discard the medication in the presence of a witness and chart the action
D. Call the physician with the patient’s refusal to take the prescribed medication
Answer : C

Question No : 80 –

A patient who sustained a chest injury has a chest tube inserted which is connected to an
under water seal drainage system. When caring for this patient the nurse will:

A. Instruct the patient to limit movement of the affected shoulder


B. Observe for fluctuation of the water level
C. Clamp the tube when needed
D. Administer hourly analgesia
Answer : B

Question No : 81 –

Which of the following laboratory blood values is expected to be decreased in hepatic


dysfunction?

A. Albumin
B. Bilirubin
C. Ammonia
D. ALT and AST
Answer : A

Question No : 82 –

A patient with allergic rhinitis reports severe nasal congestion, sneezing, and watery eyes
at various times of the year. To teach the patient to control these symptoms the nurse
advises the patient to:

A. Avoid all over the counter intranasal sprays


B. Limit the use of nasal decongestant sprays to 10 days
C. Use oral decongestants at bedtime to prevent symptoms during the night
D. Keep a diary of when an allergic reaction occurs and what precipitates it
Answer : D

Question No : 83 –

The apical pulse can be best auscultated at the:

A. Left 2nd intercostal space lateral to the mid clavicular line


B. Left 2nd intercostal space at the left sternal border
C. Left 5th intercostal space at the mid clavicular line
D. Left 5th intercostal space at the mid axillary line
Answer : C
Question No : 84 –

The nurse notes that there are no physician’s orders regarding Fatima’s post operative
daily insulin dose. The most appropriate action by the nurse is to:

A. Withhold any insulin dose since none is ordered and the patient is NPO
B. Call the physician to clarify whether insulin should be given and at what dose
C. Give half the usual daily insulin dose since she will not be eating in the morning
D. Give the patient her usual daily insulin dose since the stress of surgery will increase her blood glucose
Answer : B

Question No : 85 –

An 8-month-old infant is diagnosed with communicating hydrocephalus. The nurse notices


that his intracranial pressure is increasing from the following changes in his vital signs:

A. Bradycardia, hypotension and hypothermia


B. Bradycardia, hypertension and hyperthermia
C. Tachycardia, hypotension and hyperthermia
D. Tachycardia, hypertension and hypothermia
Answer : B

Question No : 86 –

Whenever a child with thalassemia comes for blood transfusion, he is administered


Desferoxamine (Desferal). The action of this drug is to:

A. Inhibit the inflammatory process


B. Enhance iron excretion
C. Antagonize the effect of vitamin C
D. Increase red blood cell production
Answer : B

Question No : 87 –

A patient becomes angry and threatens to leave the hospital unless the physician reviews
the reason for the patient’s delay in discharge. The patient has a medication order for
agitation available p.m..but refuses the medication and requests a drink of orange juice
instead. What should the nurse do?

A. Secretly slip the p.r.n. medication into the orange juice and give it to the patient
B. Give the patient the orange juice and tell the patient that a staff member is attempting to call the physician
C. Inform the patient that staff is unable to force anyone to stay in the hospital
D. Inform the patient that nothing can be done until the morning
Answer : B

Question No : 88 –

A nurse prepares to set up a secondary intravenous (IV) cannula. The primary IV infusing
is normal saline. In order for the secondary cannula to infuse correctly, the nurse should set
up the primary IV to:

A. Hang higher than the secondary IV


B. Hang at the same level as the secondary IV
C. Hang lower than the secondary IV
D. Discontinue before the secondary IV starts
Answer : C

Question No : 89 –

A 21 year old woman is being treated for injuries sustained in a car accident. The patient
has a central venous pressure (CVP) line insitu. The nurse recognizes that CVP
measurements:

A. Estimate Cardiac output


B. Assess myocardial workload
C. Determine need for fluid replacement
D. Determine ventilation – perfusion mismatch
Answer : C

Question No : 90 –

After application of a cast in the upper extremity, the patient complains of severe pain in the
affected site. Which of the following would the nurse initiate?
A. Administer analgesics as ordered
B. Assess neurovascular status
C. Notify his physician
D. Pad the edges of the cast
Answer : B

Question No : 96 –

The best dietary advice a nurse can give to a woman diagnosed with mild pregnancy-
induced hypertension is to:

A. Follow a strict low salt diet


B. Restrict fluid intake
C. Increase protein intake
D. Maintain a well-balanced diet
Answer : D

Question No : 97 –

A nurse is making rounds taking vital signs. Which of the following vital signs is abnormal?

A. 11 year old male – 90 b.p.m, 22 resp/min. , 100/70 mm Hg


B. 13 year old female – 105 b.p.m., 22 resp/min., 105/60 mm Hg
C. 5 year old male- 102 b.p.m, 24 resp/min., 90/65 mm Hg
D. 6 year old female- 100 b.p.m., 26 resp/min., 90/70mm Hg
Answer : B

Question No : 98 –

Which of the following actions is the most appropriate when the nurse is responding to a
patient during a tonic-clonic seizure?

A. Restrain the patient


B. Protect the patient from harm
C. Minimize noise and light stimulus
D. Apply oxygen by mask or nasal cannula
Answer : B

Question No : 99 –

The patient’s pre-operative blood pressure was 120/68 mmHg. On admission to the Post
Anesthesia Care Unit, the blood pressure was 124/70 mmHg. Thirty minutes after
admission, the patient’s blood pressure falls to 112/60 mmHg, pulse to 72 BPM, and the
skin appears warm and dry. The most appropriate action by the nurse at this time is to:

A. Raise the head of the bed


B. Notify the anesthetist immediately
C. Increase the rate of IV fluid replacement
D. Continue to monitor the patient
Answer : D

Question No : 100 –

An 84-year-old man has arthritis and is admitted for a severely edematous knee. The
physician orders heat packs every 2 hours and you feel this order may worsen the tissue
congestion. An appropriate nursing action would be:

A. Contact the physician and discuss your concerns about the order
B. To include the order in the nursing care plan and monitor outcome
C. Complete an incident report form and document concerns in the nursing notes
D. Involve the patient by asking what his treatment preference is
Answer : A

Question No : 101 –

The nurse plans the care for a patient with increased intracranial pressure, she knows that
the best way to position the patient is to:
A. Keep patient in a supine position until stable
B. Elevate the head of the bed to 30 degrees
C. Maintain patient on right side with head supported on a pillow
D. Keep patient in a semi-sitting position
Answer : B

Question No : 102 –

The coronary care nurse draws an Arterial Blood Gas (ABG) sample to assess a patient for
acidosis. A normal pH for arterial blood is:

A. 7.0 – 7.24
B. 7.25 – 7.34
C. 7.35 – 7.45
D. 7.5 – 7.6
Answer : C

Question No : 103 –

A patient voided a urine specimen at 9:00 AM. The specimen should be sent to the
laboratory before:

A. 9:30 AM
B. 10:00 AM
C. 10:30 AM
D. 11:00 AM
Answer : A

Question No : 104 –

Which of the following correctly describes wound packing in a wet to dry dressing?

A. Pack gauze into the wound tightly


B. Overlap the wound edges with wet packing
C. Pack the wound with slightly moistened gauze
D. Use gauze well saturated with saline for packing the wound
Answer : C

Question No : 105 –

To prevent post-operative thrombophlebitis, which one of the following measures is


effective?

A. Elevation of the leg on two pillows


B. Using of compression stocking at night
C. Massage the calf muscle frequently
D. Performing leg exercises
Answer : D

Question No : 106 –

The mother of a child with nephrotic syndrome asks why her child must be weighed each
morning. The nurse’s response should be based on the fact that this is important to
determine the:

A. Nutritional status
B. Water retention
C. Medication doses
D. Blood volume
Answer : B

Question No : 107 –

When caring for a patient with hepatic encephalopathy the nurse may carry out the
following orders: give enemas, provide a low protein diet, and limit physical activities.
These measures are performed to:

A. Minimize edema
B. Decrease portal pressure
C. Reduce hyperkalemia
D. Decrease serum ammonia
Answer : D
Question No : 108 –

A patient is to receive 2.5mg of morphine sulfate. The ampoule contains l000mcg/mL. How
much morphine should the nurse administer?

A. 0.25 ml
B. 1 ml
C. 1.5 ml
D. 2.5 ml
Answer : D

Question No : 109 –

When the post-operative patient returns to the surgical unit, the priority is to:

A. Assess the patient’s pain


B. Measure the patient’s vital signs
C. Monitor the rate of the IV infusion
D. Check the physician’s post-operative orders
Answer : B

Question No : 110 –

While preparing the midday medications, the nurse finds difficulty reading the label on a
medicine bottle. The best action by the nurse is to:

A. Document that it could not be given due to difficulty in reading the label
B. Make out a new label with clear handwriting using adhesive tape to attach it
C. Ask the pharmacist to replace it with a clearly labeled bottle
D. Give the medication if it is similar to a bottle present on the trolley
Answer : C

Question No : 111 –

To ensure safe practice during defibrillation, the nurse must:


A. Cover paddles with electrode gel
B. Avoid touching the patient’s bed
C. Remove paddles after the shock
D. Synchronize prior to shock delivery
Answer : B

Question No : 112 –

The physician orders heparin 40 000 U in 1 liter of D5W IV to infuse at 1000 U/hr. What is
the flow rate in milliliters per hour?

A. 250 mls/hr
B. 25 mls/hr
C. 2.5 mls/hr
D. 0.25 mls/hr
Answer : B

Question No : 113 –

What two behaviors are important when documenting the depth of the patients
depression?

A. Orientation and appearance


B. Helplessness and hopelessness
C. Affect and thought processes
D. Mood and impulse control
Answer : B

Question No : 114 –

The nurse knows that the greatest risk for a patient with a ruptured ectopic pregnancy is:

A. Hemorrhage leading to hypovolemic shock


B. Strictures and scarring of the fallopian tube
C. Adhesions and scarring from blood in the abdomen
D. Infertility resulting from treatment with a salpingectomy
Answer : A
Question No : 115 –

The nurse manager has just prepared a medication for a patient and she asked you to give
the medication. Which of the following is the best response to the nurse manager’s
request?

A. Give the patient his medication and record it on the chart


B. Ask another nursing colleague to give and record the medication
C. Explain that you cannot give a medication that you did not prepare
D. Give the medication and ask the nurse manager to record it on the chart
Answer : C

Question No : 116 –

A patient presents to the emergency department with diminished and thready pulses,
hypotension and an increased pulse rate. The patient reports weight loss, lethargy, and
decreased urine output. The lab work reveals increased urine specific gravity. The nurse
should suspect:

A. Renal failure
B. Sepsis
C. Pneumonia
D. Dehydration
Answer : D

Question No : 117 –

A patient is to receive 50mL of fluid in 1/2 hour (30 min). The infusion pump should be set
to deliver how many milliters per hour?

A. 25 ml/hr
B. 50 ml/hr
C. 75 ml/hr
D. 100 ml/hr
Answer : D
Question No : 118 –

A patient with a history of angina pectoris arrives in emergency complaining of headache,


visual disturbances and feeling dizzy. Your nursing assessment also notes he looks
flushed, is perspiring perfusely and is experiencing palpitations. You should suspect:

A. An overdose of sublingual nitroglycerin


B. The onset of a myocardial infarction
C. The patient has been over exercising
D. The beginning of a severe chest infection
Answer : A

Question No : 119 –

The purpose of a cardiac pacemaker is to:

A. Initiate and maintain the heart rate when SA node is unable to do so


B. Stabilize the heart rate when it is above 100 beats per minute
C. Stabilize the heart when the patient has had a heart attack
D. Regulate the heart when the patient is going for open heart surgery
Answer : A

Question No : 120 –

A nurse has been working in a general hospital on the same medical unit for 6 years. The
Behavioral Unit is desperately short staffed and the nurse is asked to work her shift in this
other unit. What would be the expected response of the nurse to this request?

A. “I will go to the unit and hopefully the behavioral health staff members will assist me with my assignment.”
B. “I cannot go. I have no previous behavioral health experience. I do not want to reduce the quality of patient
care.”
C. “I have no previous behavioral health experience. I am willing to go and help with any duties that are similar
to those I perform on my unit.”
D. “I should not be expected to float to another unit without a proper orientation. I will fill out an incident
report if I am sent there.”
Answer : C

Question No : 121 –

Order: Allopurinol 450 mg p.o. daily. Drug availablE.Allopurinol 300 mg scored tablets.
Which of the following will you administer?

A. 0.5 tablet
B. 1.5 tablets
C. 2 tablets
D. 2.5 tablets
Answer : B

Question No : 122 –

The correct way to trim the toe nails of a patient with diabetes is to:

A. Cut the nails in a curve and then file


B. Cut the nails straight across and then file
C. File the nails straight across and square only
D. File the nails in a curved arch with low sides only
Answer : C

Question No : 123 –

A patient complains of pain in his right arm. The physician orders codeine 45 mg and
aspirin 650 mgs every 4 hours PRN. Each codeine tablet contains 15mg of codeine and
each aspirin tablet contains 325mg of aspirin. What should the nurse administer?

A. 2 codeine tablets and 4 aspirin tablets


B. 4 codeine tablets and 3 ½ aspirin tablets
C. 3 codeine tablets and 2 aspirin tablets
D. 3 codeine tablets and 3 aspirin tablets
Answer : C

Question No : 124 –

During the acute phase of a cerebrovascular accident (CVA) the nurse should maintain the
patient in which of the following positions?

A. Semi-prone with the head of the bed elevated 60-90 degrees


B. Lateral, with the head of the bed flat
C. Prone, with the head of the bed flat
D. Supine, with the head of the bed elevated 30-45 degrees
Answer : D

Question No : 125 –

A patient on diuretics has had vomiting and diarrhea for the past 3 days. Which of the
following is this patient most at risk for developing?

A. Hypokalemia and cardiac arrhythmias


B. Hypercalcemia and polyuria
C. Dehydration and hyperglycemia
D. Hyperalimentation and heart block
Answer : A

Question No : 126 –

A patient admitted to the hospital in hypertensive crisis is ordered to receive hydralazine


(Apresoline) 20mg IV stat for blood pressure greater than 190/100 mmHg. The best
response of the nurse to this order is to:

A. Give the dose immediately and once


B. Give medication if patient’s blood pressure is > 190/100 mmHg
C. Call the physician because the order is not clear
D. Administer the dose and repeat as necessary
Answer : A

Question No : 127 –

Which of the following is the most appropriate first action for the nurse to take for a pre-
schooler who has fallen and has a hematoma formed on the temporal bone?

A. Encourage a nap
B. Give pain medication
C. Apply ice and monitor vital signs
D. Allow the child to continue activities
Answer : C

Question No : 128 –

The minimal amount of urine that a post-operative patient weighing 60 kgs should pass is?

A. 120 ml/hr
B. 90 ml/hr
C. 60 ml/hr
D. 30 ml/hr
Answer : D

Question No : 129 –

You are the nurse providing post-operative care for a 9 year old boy who is 6 hours post-
tonsillectomy. He is sleeping, but on routine observation you notice that his pulse has
increased, he seems to be restless and trying to swallow frequently. Your immediate action
would be:

A. Apply an ice collar to reduce discomfort


B. Allow child to keep sleeping and record observations
C. Wake him and offer some ice chips to suck
D. Check inside his mouth for any signs of bleeding and notify the physician
Answer : D

Question No : 130 –

When inserting a rectal suppository for a patient the nurse should?

A. Insert the suppository while the patient performs the ‘valsava maneuver’
B. Place the patient in a supine position
C. Position the suppository beyond the muscle sphincter of the rectum
D. Insert the suppository 1/2 inch into the rectum
question_answerVIEW ANSWER

Answer : C

Question No : 131 –

A patient with pneumonia is coughing up purulent thick sputum. Which one of the following
nursing measures is most likely helpful to loosen the secretions?

A. Postural drainage
B. Breathing humidified air
C. Percussion over the affected lung
D. Coughing and deep breathing exercises
Answer : B

Question No : 132 –

A patient is admitted to a hospital with acute renal failure. The patient wakes up
complaining of abdominal pain. On assessment, the nurse observes edema to the patient’s
ankles and distended neck veins. The patient is dyspneic with a blood pressure of 200/96
mmHg. The proper nursing diagnosis for this patient is:

A. Deficient fluid volume related to disease process


B. Excess fluid volume related to decreased glomerular filtration rate
C. Knowledge deficit related to proper medication regimen
D. Acute pain related to renal edema
Answer : B

Question No : 133 –

A patient arrives at the emergency department with slurred speech and a right facial droop.
The patient’s relative states the patient “is not himself.” Upon assessment, the nurse finds
paresthesia to the right side of the body, receptive aphasia, hemianopia and altered
cognitive abilities. The nurse should suspect:
A. A narcotic overdose
B. Parkinson’s disease
C. Alcohol withdrawal
D. A cerebrovascular accident (CVA)
Answer : D

Question No : 134 –

The nurse is preparing teaching plans for several patients. The nurse should recognize
which of the following patients is at greatest risk for fluid and electrolyte imbalance?

A. A 2-year-old patient who is receiving gastrostomy feedings


B. A 20-year-old patient with a sigmoid colostomy
C. A 40-year-old patient who is 3 days post-operative with an ileostomy
D. A 60-year-old patient who is 8 hours post-renal arteriography
Answer : C

Question No : 135 –

A 3-month-old infant is admitted with a diagnosis of ventricular septal defect. The physical
assessment for this infant would reveal:

A. High pitched cry


B. Harsh heart murmur
C. Bradycardia
D. Hypertension
Answer : B

Question No : 136 –

A young patient is extremely irritable due to meningitis. It would be most important for the
nurse to:

A. Use low-level lighting in the room


B. Ventilate the room
C. Eliminate strong odors
D. Allow frequent visitors
Answer : A

Question No : 137 –

A male patient with a right pleural effusion noted on a chest X-ray is being prepared for
thoracentesis. The patient experiences severe dizziness when sitting upright. To provide a
safe environment, the nurse assists the patient to which position for the procedure?

A. Prone with head turned toward the side supported by a pillow


B. Sims’ position with the head of the bed flat
C. Right side-lying with the head of the bed elevated 45 degrees
D. Left side-lying with the head of the bed elevated 45 degrees
Answer : D

Question No : 138 –

A newborn is diagnosed with ventricular septal defect. The baby is discharged with a
prescription for digoxin syrup 20 micrograms bid. The bottle of digoxin is labeled as 0.05
mg/ml. The nurse should teach the mother to administer on each dose:

A. 0.1 ml
B. 0.2 ml
C. 0.4 ml
D. 0.8 ml
Answer : C

Question No : 139 –

As a part of the treatment given to a child with leukemia the child is placed on reverse
barrier isolation to:

A. Protect the child from injury


B. Protect the child from infectious agents
C. Provide the child with a quiet environment
D. Keep the child away from other children
Answer : B

Question No : 140 –

The nurse should be aware that tetracycline is contraindicated in children under 12 years of
age because:

A. Minimal doses are needed to control infection


B. Immunosuppression is a common side effect
C. Staining of the teeth is an adverse effect
D. They are prone to develop renal failure
Answer : C

Question No : 146 –

When caring for a patient with acute pancreatitis, the patient is most likely to complain of
pain which is:

A. Severe and located in the left lower quadrant and radiating to the groin
B. Burning and located in the epigastric area and radiating to the groin
C. Severe and located in the epigastric area and radiating to the back
D. Burning and located in the left lower quadrant and radiating to the back
Answer : C

Question No : 147 –

The best time for the nurse to teach an anxious patient about the patient controlled
analgesic (PCA) pump would be during which of the following stages of patient care?

A. Post-operative
B. Pre-operative
C. Intraoperative
D. Post anesthesia
Answer : B

Question No : 148 –

Elevated levels of amylase and lipase in the blood are common in:
A. Diabetes mellitus
B. Esophagitis
C. Pancreatitis
D. Hepatitis
Answer : C

Question No : 149 –

In preparing the patient for electroencephalogram (EEG), the nurse should:

A. Withhold breakfast
B. Give sleeping pills the night before
C. Shave the hair
D. Restrict intake of coffee
Answer : D

Question No : 150 –

An 11 year old girl with a history of asthma arrives at the primary health clinic with
signs/symptoms of shortness of breath, audible wheezing, nasal flaring and mild lip
cyanosis. Your immediate nursing action is to:

A. Assess respiratory distress and peak expiratory flow rate


B. Take a blood sample to assess COlevels
C. Instruct the parents to take the child immediately to hospital
D. Sit the child comfortably and offer 2 puffs of ventolin stat
Answer : A

Question No : 141 –

While assessing an 84-year-old post-operative patient, the nurse observes that the patient
suddenly becomes very anxious, appears cyanotic and has severe dyspnea. The nurse
recognizes these symptoms to be consistent with:

A. Congestive heart failure


B. Pulmonary embolism
C. COPD exacerbation
D. Myocardial infarction
Answer : B

Question No : 142 –
When preparing an eye medication, the nurse reads the order “OS”. Medication is given
into:

A. Both eyes
B. Left eye
C. Right eye
D. Infected eye
Answer : B

Question No : 143 –

A patient has been taking Aluminum Hydroxide daily for 3 weeks. The nurse should be alert
for which of the following side effects?

A. Constipation
B. Flatulence
C. Nausea
D. Vomiting
Answer : A

Question No : 144 –

An early sign of acute respiratory failure is:

A. Diaphoresis
B. Cyanosis
C. Restlessness
D. Orthopnea
Answer : C

Question No : 145 –

In caring for a woman and baby day 3 postnatally, she tells you that her baby has not had a
bowel action since delivery. Your appropriate response would be:

A. Reassure the mother that it is quite normal for a baby to not move their bowels until day 5 after a few days
of milk feeding
B. Start a bowel chart, document all findings, and wait another 48 hours before reporting to the physician
C. Encourage more frequent warm baths for the neonate with gentle abdomen massages
D. Tell the mother that you will let the physician know, so the baby can be checked for any obstruction
Answer : D

1. The nurse is teaching a mother whose daughter has iron deficiency anemia. The nurse determines the parent
understood the dietary modifications, if she selects?

a) Bread and coffee


b) Fish and Pork meat
c) Cookies and milk
d) Oranges and green leafy vegetables

2. Which of the following is the most common clinical manifestation of G6PD following ingestion of aspirin?

a) Kidney failure
b) Acute hemolytic anemia
c) Hemophilia A
d) Thalassemia

3. The nurse assesses a client with an ileostomy for possible development of which of the following acid-base
imbalances?

a) Respiratory acidosis
b) Metabolic acidosis
c) Metabolic alkalosis
d) Respiratory alkalosis

4. The nurse anticipates which of the following responses in a client who develops metabolic acidosis.

a) Heart rate of 105 bpm


b) Urinary output of 15 ml
c) Respiratory rate of 30 cpm
d) Temperature of 39 degree Celsius

5. A client has a phosphorus level of 5.0mg/dL. The nurse closely monitors the client for?

a) Signs of tetany
b) Elevated blood glucose
c) Cardiac dysrhythmias
d) Hypoglycemia

6. A nurse is caring for a child with pyloric stenosis. The nurse would watch out for symptoms of?

a) Vomiting large amounts


b) Watery stool
c) Projectile vomiting
d) Dark-colored stool

7. The nurse responder finds a patient unresponsive in his house. Arrange steps for adult CPR.

a) Assess consciousness
b) Give 2 breaths
c) Perform chest compression
d) Check for serious bleeding and shock
e) Open patient’s airway
f) Check breathing
___, ___, ___, ___, ___, ___

8. Which of the following has mostly likely occurred when there is continuous bubbling in the water seal chamber of the
closed chest drainage system?

a) The connection has been taped too tightly


b) The connection tubes are kinked
c) Lung expansion
d) Air leak in the system

9. Which if the following young adolescent and adult male clients are at most risk for testicular cancer?

a) Basketball player who wears supportive gear during basketball games


b) Teenager who swims on a varsity swim team
c) 20-year-old with undescended testis
d) Patient with a family history of colon cancer

10. The nurse plans to frequently assess a post-thyroidectomy patient for?

a) Polyuria
b) Hypoactive deep tendon reflex
c) Hypertension
d) Laryngospasm

11. An 18-month-old baby appears to have a rounded belly, bowlegs and slightly large head. The nurse concludes?

a) The child appears to be a normal toddler


b) The child is developmentally delayed
c) The child is malnourished
d) The child’s large head may have neurological problems.

12. A nurse is going to administer 500mg capsule to a patient. Which is the correct route?

A.)

B.)
C.)

D.)

13. An appropriate instruction to be included in the discharge teaching of a patient following a spinal fusion is?

a) Don’t use the stairs


b) Don’t bend at the waist
c) Don’t walk for long hours
d) Swimming should be avoided

14. A nurse is preparing to give an IM injection of Iron Dextran that is irritating to the subcutaneous tissue. To prevent
irritation to the tissue, what is the best action to be taken?

a) Apply ice over the injection site


b) Administer drug at a 45 degree angle
c) Use a 24-gauge-needle
d) Use the z-track technique

15. What should a nurse do prior to taking the patient’s history?

a) Offer the patient a glass of water


b) Establish rapport
c) Ask the patient to disrobe and put on gown
d) Ask pertinent information for insurance purposes

16. A pregnant woman is admitted for pre-eclampsia. The nurse would include in the health teaching that magnesium
will be part of the medical management to accomplish the following?

a) Control seizures
b) promote renal perfusion
c) To decrease sustained contractions
d) Maintain intrauterine homeostasis

17. A nurse is going to administer ear drops to a 4-year-old child. What is the correct way of instilling the medicine after
tilting the patient’s head sidewards?

a) Pull the pinna back then downwards


b) Pull the pinna back then upwards
c) Pull the pinna up then backwards
d) Pull the pinna down then backwards

18. A nursing student was intervened by the clinical instructor if which of the following is observed?

a) Inserting a nasogastric tube


b) Positioning the infant in a “sniffing “position
c) Suctioning first the mouth, then the nose
d) Squeezing the bulb syringe to suction mouth

19. Choose amongst the options illustrated below that best describes the angle for an intradermal injection?

A.)

B.)

C.)

D.)
20. During a basic life support class, the instructor said that blind finger sweeping is not advisable for infants. Which
among the following could be the reason?

a) The mouth is still too small


b) The object may be pushed deeper into the throat
c) Sharp fingernails might injure the victim
d) The infant might bite

21. A nurse enters a room and finds a patient lying on the floor. Which of the following actions should the nurse perform
first?

a) Call for help


b) Establish responsiveness of patient
c) Ask the patient what happened
d) Assess vital signs

22. A patient with complaints of chest pain was rushed to the emergency department. Which priority action should the
nurse do first?

a) Administer morphine sulfate intravenously


b) Initiate venous access by performing venipunture
c) Administer oxygen via nasal cannula
d) Complete physical assessment and patient history

23. A rehab nurse reviews a post-stroke patient’s immunization history. Which immunization is a priority for a 72-year-old
patient?

a) Hepatitis A vaccine
b) Hepatitis B vaccine
c) Rotavirus Vaccine
d) Pneumococcal Vaccine

24. Several patients from a reported condominium fire incident were rushed to the emergency room. Which should the
nurse attend to first?

a) A 15-year-old girl, with burns on the face and chest, reports hoarseness of the voice
b) A 28-year-old man with burns on all extremities
c) A 4-year-old child who is crying inconsolably and reports severe headache
d) A 40-year-old woman with complaints of severe pain on the left thigh

25.The doctor ordered 1 pack of red blood cells (PRBC) to be transfused to a patient. The nurse prepares the proper IV
tubing. The IV tubing appropriate for blood transfusion comes with?

a) Air vent
b) Microdrip chamber
c) In-line filter
d) Soluset

26. The expected yet negative (harmful ) result for posthemodialysis is a decrease in?

a) Creatinine
b) BUN
c) Phosphorus
d) Red blood cell count
27. A patient was brought to the emergency room after she fell down the stairs. Which of the following is the best
indicator for increased intracranial pressure in head and spinal injury?

a) Inability to move extremities


b) Decreased respiratory rate
c) Increase in pulse and blood pressure
d) Decrease level of consciousness

28. A new nurse is administering an enema to a patient. The senior nurse should intervene if the new nurse?

a) Hangs the enema bag 18 inches above the anus


b) Positions the client on the right side
c) Advances the catheter 4 inches into the anal canal
d) Lubricates 4 inches of the catheter tip

29. The medication nurse is going to give a patient his morning medications. What is the primary action a nurse should
do before administering the medications?

a) Provide privacy
b) Raise head of the bed
c) Give distilled water
d) Check client’s identification bracelet

30. A 30-year-old client is admitted with inflammatory bowel syndrome (Crohn’s disease). Which of the following
instructions should the nurse include in the health teaching? Select all that apply

a) Corticosteroid medication is part of the treatment


b) Include milk in the diet
c) Aspirin should be administered
d) Antidiarrheal medication can help

1.) Answer: D

Dark green leafy vegetables are good sources of iron. Oranges are good sources of vitamin C that enhances iron
absorption in the small intestines.

2.) Answer: B
Individuals with G6PD may exhibit hemolytic anemia when exposed to infection, certain medications or chemicals.
Salicylates such as Aspirin damages plasma membranes of erythrocytes, leading to hemolytic anemia.

3.) Answer: B

Lower GI fluids are alkaline in nature and can be lost via ileostomy. Thus, loss of HCO3, results to metabolic acidosis.

4.) Answer: C

Initially, respiratory system will try to compensate metabolic acidosis. Patients with metabolic acidosis have high
respiratory rate.

5.) Answer: A

Normal phosphorus level is 2.5 – 4.5 mg/dL .The level reflects hyperphosphatemia which is inversely proportional to
calcium. Client should be assessed for tetany which is a prominent symptom of hypocalcemia.

6.) Answer: C

Clinical manifestations of pyloric stenosis include projectile vomiting, irritability, constipation, and signs of dehydration,
including a decrease in urine output.

7.) Answer: A, E, F, C, B, D

In accordance with the new guidelines, remember AB-CABS. A-airway B-breathing normally? − C-chest compression A-
airway open B-breathing for patient S-serious bleeding, shock, spinal injury. The nurse should first assess consciousness
of the patient. Next, open patient’s airway to check for breathing. When there is no breathing, immediately perform
chest compression then give 2 breaths, do the cycle of care over. Finally, check for serious bleeding, shock, and spinal
injury.

8.) Answer: D

Continuous bubbling seen in water-seal bottle/ chamber indicates an air leak or loose connection, and air is sucked
continuously into the closed chest drainage system.

9.) Answer: C

Testicular cancer is most likely to affect males in late adolescence. Undescended testis is also one major risk for testicular
cancer.

10.) Answer: D

Hypocalcemia occurs when there is accidental removal or destruction of parathyroid tissue during surgical removal of the
thyroid gland. Laryngospasm is one of the clinical manifestations of tetany, an indicator of hypocalcemia.

11.) Answer: A

It’s normal for a toddler to have bowlegs and a protruding belly. The head still appears somewhat large in proportion
from the rest of the body.

12.) Answer: D

13.) Answer: B

There is 6-8 months activity restriction following a spinal fusion. Sitting, lying, standing, normal stair climbing, walking,
and gentle swimming is allowed. Bending and twisting at the waist should be avoided, along with lifting more than 10
lbs.

14.) Answer: D
Z-track technique is used to administer drugs especially irritating to the subcutaneous tissue. This method promotes
absorption of the drug by preventing drug leakage into the subcutaneous layer.

15.) Answer: B

Establishing rapport is a way to gain trust that will lead for a patient to relax. You can get more insights and information
from a patient when rapport is established.

16.) Answer: A

Low magnesium (hypomagnesemia) produces clinical manifestations like increased reflexes, tremors, and seizures.
Magnesium Sulfate is the drug of choice to prevent seizures in pre-eclampsia and eclampsia.

17.) Answer: C

Ear canal of children ages 3years and above can be straightened by pulling the pinna up then backwards. For children
below 3 years of age, the ear canal can be straightened by pulling the pinna down then backwards.

18.) Answer: A

Infants are nose breathers. A gastric tube may be inserted to facilitate lung expansion and stomach decompression, but
not a nasogastric tube as it can occlude the nare, thus, making breathing difficult for the infant.

19.) Answer: B

20.) Answer: B

Blind finger sweeps are not recommended in all CPR cases especially for infants and children because the foreign object
may be pushed back into the airway.

21.) Answer: B

First step in cardiopulmonary resuscitation (CPR) is assessing responsiveness of the patient.

22.) Answer: C

Priority nursing action is to administer oxygen to patients with chest pain. Chest pain is caused by insufficient myocardial
oxygenation.

23.) Answer: D

Pneumococcal Vaccine is a priority immunization for the elderly. Seniors, ages 65 years old and above, have higher risk
for serious pneumococcal infection and likely have low immunity. This is administered every 5 years.

24.) Answer: A

Burns on the face and neck can cause swelling of the respiratory mucosa that can lead to airway obstruction manifested
by hoarseness of voice and difficulty in breathing. Maintaining an airway patency is the main concern.

25.) Answer: C

An in-line filter is required for blood transfusions.

26.) Answer: D

Negative outcome: Hemodialysis decreases red blood cell count which worsens anemia, because RBCs are lost in dialysis
from anticoagulation during the procedure, and from residual blood that is left in the dialyzer.

27.) Answer: D
Decrease in level of consciousness and headache are early signs of increase in intracranial pressure (ICP). Altered level of
consciousness is the most common symptom that indicates a deficit in brain function.

28.) Answer: B

Recall the anatomy of the colon. The appropriate position is left lateral to facilitate flow of enema by gravity into the
colon.

29.) Answer: D

Recall the 12 Rights of administration. Checking the patient’s name is critical for client-safety.

30.) Answer: A , D

Crohn’s disease is a chronic inflammation of the colon with symptoms of diarrhea, abdominal pain, and weight loss.
Corticosteroid is a treatment for Crohn’s disease. Antidiarrheal can give relief to diarrheal episodes. Aspirin should be
avoided as it can worsen inflammation. Those with Crohn’s disease are mostly lactose intolerant, so choice no. (2) is
incorrect.

1.The client presented with complaints of body weakness, dizziness and chest pain. Upon careful assessment, the nurse
suspects Angina Pectoris. Which of the following statements made by the client can confirm this?

a. “I suddenly felt a pain on my chest which radiates to my back and arms”.


b. “I suddenly felt a sharp pain on my lower abdomen”.
c. “The pain does not subside even if I rest”.
d. “The pain goes all the way down to my stomach”.

2.The client from the OR is transferred to the post-anesthesia care unit after surgical repair of abdominal aortic
aneurysm. Which of the following assessment findings would indicate that the repair was successful?

a. Urine output of 50 mL/hr.


b. Presence of non-pitting, peripheral edema.
c. Clear sclera.
d. Presence of carotid bruit.

3.The client is scheduled for cardiac catheterization because the physician wants to view the right side of the heart.
Which of the following would the nurse expect to see in this procedure?

a. A dye is injected to facilitate the viewing of the heart


b. Thallium is injected to facilitate the scintillation camera
c. A probe with a transducer tip is swallowed by the client.
d. A tiny ultrasound probe is inserted into the coronary artery

4.The client is being treated for hypovolemia. To assess the effectiveness of the treatment, the Central Venous Pressure
(CVP) of the client is being monitored. Which of the following is TRUE about CVP?

a. The CVP is measured with a central venous line in the inferior vena cava.
b. The normal CVP is 7 to 9 mmHg.
c. The zero point on the transducer needs to be at the level of the left atrium.
d. The client needs to be supine, with the head of the bed elevated at 45 degrees.

5.The client’s ECG tracing shows ventricular tachycardia secondary to low magnesium level. Which of the following
electrocardiogram tracing results is consistent with this finding?
a. The appearance of a U wave
b. Shortened ST segment and a widened T wave.
c. Tall, peaked T waves
d. Tall T waves and depressed ST segment

6.The nurse is teaching the client how to use a dry powder inhaler (DPI). Which of the following are correct instructions
given by the nurse? Select All That Apply.

a. Load the drug first by turning to the next dose of drug, or inserting the capsule into the device, or inserting the disk or
compartment into the device.
b. Never wash or place the inhaler in water.
c. Shake your inhaler prior to use.
d. The drug is a dry powder that is why you will taste the drug as you inhale.
e. Never exhale into the inhaler.
f. Do not remove the inhaler from your mouth as soon as you have breathe in.

7.The nurse is assigned to render care for a client who has a chest tube drainage system. Which of the following are
appropriate nursing actions? Select All That Apply.

a. Strip the chest tube.


b. Empty collection chamber when the drainage makes contact to the bottom of the tube.
c. Keep chest tube as straight as possible.
d. Notify the physician of drainage is greater than 70mL/hr.
e. Assess bubbling in the water seal chamber.
f. Keep drainage system at the level of the client’s chest

8.The client with DKA is receiving bicarbonate IV infusion for treatment of severe metabolic acidosis. The nurse notes
that the latest ABG shows a pH of 7.0. What should the nurse keep in mind in giving the drug?

a. Check vital signs before giving the drug and monitor serum sodium level.
b. Perform a sensitivity test prior to drug administration.
c. Mix the drug with D10W 500 ml IV fluid and infuse for over 4 to 8 hours.
d. Administer the drug slowly and monitor the potassium leve

9.The client with a gunshot wound on the abdomen starts to get lethargic, is breathing heavily, and the wound dressing is
fully soaked with blood. The nurse is expected to immediately perform which of the following actions?

a. Loosen tight clothing and administer oxygen supply.


b. Apply warm blanket to prevent heat loss.
c. Apply large gauze on the bleeding site to put direct pressure or place a tourniquet on the artery near the bleeding site.
d. Initiate IV access.

10.The nurse is providing home instructions to a client with increased adrenocorticotrophic hormone. The nurse is aware
that the client with excessive corticosteroids is suffering from what condition?

a. Cushing’s syndrome
b. Addison’s disease
c. Hypothyroidism
d. SIADH

11.The nurse is assigned to a post-thyroidectomy client and is monitoring for signs of hypocalcemia. The nurse gently
tapped the area below the zygomatic bone just in front of the ear. This action will elicit:

a. Facial tremor
b. Hyperreflexia
c. Chvostek sign
d. Trousseau sign

12.The nurse is caring for a client with an antineoplastic IV hooked on the right hand. The nurse notices that IV site is
swelling and feels cool when touched. The nurse recognizes this as extravasation. This predisposes the client to develop
which among the following complications? Select all that apply.

a. Infection
b. Tissue necrosis
c. Disfigurement
d. Loss of function
e. Amputation
f. Delayed healing

13.Nursing interventions commonly performed when the client is experiencing Autonomic Dysreflexia will include the
following. Select all that apply.

a. Use digital stimulation to empty the bowel.


b. Have the client sit up straight and raise his head so that he is looking ahead.
c. Remove client’s stockings or socks.
d. Manually compress or tap the bladder to allow urine to flow down the catheter.
e. Administer prescribed vasodilators.

14.Neurologic conditions can be manifested by changes in breathing patterns. The client presents with symptom of
Cheyne-Stokes respirations. The nurse knows that this kind of breathing pattern shows:

a. Completely irregular breathing pattern with random deep and shallow respirations
b. Prolonged inspirations with inspiratory and /or expiratory pauses
c. Sustained regular rapid respirations of increased depth
d. Rhythmic waxing and waning of both rate and depth of respiration with brief periods of interspersed apnea

15.The physician is assessing the client’s sensorium by using the Glasgow Coma Scale. Which of the following is true
about the Glasgow Coma Scale?

a. If the client does not respond to painful stimuli, the score is 0.


b. A score lower than 10 indicates that the client is in a coma.
c. A score of 8 indicates that the client is alert and oriented.
d. A score of 4 indicates that the client sustained severe head trauma.

16.The nurse on duty is caring for a client with Amyotrophic Lateral Sclerosis and is concerned with the client’s impaired
physical mobility. The following nursing interventions are geared towards maintaining optimal physical mobility EXCEPT:

a. Maintain an exercise program.


b. Encourage participation in activities.
c. Instruct client related safety measures.
d. Schedule activities in the morning.

17.An elderly client had a cerebrovascular accident or stroke. The left brain is affected and is at risk for impaired verbal
communication. The nurse asked a question and noted that the client has difficulty talking and communicating his
thoughts. Which of the following terms should the nurse use to document the finding?

a. Receptive Aphasia
b. Expressive Aphasia
c. Global aphasia
d. Apraxia
18.The client diagnosed with Alzheimer’s disease is starting to show signs and symptoms. The nurse wants to assess for
graphesthesia. This is performed by:

a. Testing for the client’s ability to identify an object that is placed on the hand with eyes closed.
b. Testing for the client’s ability to recognize the written letter or number in the client’s skin while the eyes are closed.
c. Making the client stand, with the arms at the side, feet together, with the eyes open and then closed. The client is then
observed for any swaying.
d. Testing for the presence of pain once the leg is flexed at the hip, and then extended.

19.The pediatric client presents with the following signs and symptoms: high fever, drooling, difficulty of breathing and
leaning forward in a tripod position. Immunization history shows that the client never received any Hib vaccine. Which of
the following is the priority of the healthcare provider?

a. Continuous oxygen therapy and constant monitoring of oxygen saturation rate.


b. A well regulated IV infusion and timely administration of antibiotics.
c. Vaccination of Hib and other remaining vaccines to complete required immunizations.
d. Avoiding any throat examination or agitation of the child.

20.The physician prescribed Clarithromycin (Biaxin) 250mg BID x 7 days for the client’s infection. Incorrect drug frequency
and duration would cause inaccurate transfer time of the drug to specific tissues in the body. The nurse explains to the
client that accumulation of the drug in specific tissues is the concept of:

a. Absorption
b. Distribution
c. Metabolism
d. Excretion

21.The nurse is to administer Meperidine (Demerol) 35 mg through the intramuscular route. Available meperedine is
50mg/mL. Which of the following is the least favorable injection site for intramuscular medication?

a. Ventrogluteal
b. Vastus lateralis
c. Deltoid muscle
d. Dorsogluteal

22.The client presented with complaint of leg cramps. Upon checking the client’s chart, the nurse noted that the client is
hypertensive and is prescribed with a Thiazide diuretic. The appropriate nursing intervention for this client is:

a. Stop the Thiazide diuretic


b. Refer to the physician for evaluation of electrolyte level of the client
c. Switch the client to a loop diuretic
d. Give the client a non-steroidal anti-inflammatory drug (NSAID)

23.The client is wheeled into the delivery room and is ready for childbirth. While crowning occurs, the labor nurse applies
gentle pressure over the perineum and fetal head. The maneuver performed is called:

a. Brandt-Andrew’s maneuver
b. McRobert’s maneuver
c. Schultz mechanism
d. Ritgen’s maneuver

24.The nurse is monitoring the condition of the postpartum client. As a part of the postpartum adaptations, the nurse
monitors for descent of the uterus and expects the fundus to be:
a. On the same level after delivery
b. Decreased by 1 cm/day
c. Decreased by 1.5 cm/day
d. Decreased by 2 cm/day

25.The granddaughter of the client asked the nurse if it is normal for elderly people to feel sleepy despite sleeping for
long hours. Which of the following conditions would the nurse suspect?

a. Somatoform Disorder
b. Malingering
c. Anxiety
d. Amnesia

26.Chemotherapy is one of the treatments for uterine cancer. The client asked the nurse how chemotherapeutic drugs
work. Which of the following statements will be the best explanation?

a. Chemotherapeutic agents alter molecular structure of DNA.


b. Chemotherapeutic agents hasten cell division.
c. Cancer cells are sensitive only to chemotherapeutic agents.
d. Chemotherapeutic agents act on all rapidly dividing cells.

27.Vomiting is one of the most common side effects of chemotherapy. The nurse should be aware of which acid-base
imbalance?

a. Ketoacidosis
b. Metabolic acidosis
c. Metabolic alkalosis
d. Respiratory alkalosis

28.The client develops a 2nd degree skin reaction from radiation therapy. The nurse should expect the following
symptoms EXCEPT:

a. The skin is scaly.


b. There is an itchy feeling.
c. There is dry desquamation present.
d. The skin is reddened.

29.The nurse is assessing the muscle coordination and mobility of the client with musculoskeletal disorder. The nurse
noted impulsive and brief muscle twitching of the face and the limbs. This finding is called:

a. Tremor
b. Chorea
c. Athetosis
d. Dystonia

30.The nurse is assigned to render care to a client with altered mobility. Which of the following statements is true
regarding body mechanics when moving clients?

a. Stand at arm’s length from the working area.


b. Elevate adjustable beds to hip level.
c. Swivel the body when moving the client.
d. Move the client with wide base and straight knees.
1. Answer: A

Rationale:

Angina pectoris is a substernal pain that radiates to the neck, jaw, back and arms and is relieved by rest. Lower
abdominal pain may indicate other gastrointestinal problems.

2. Answer: A

Rationale:

50 mL/hr is the normal urine output. A normal urine output indicates that there is a good renal perfusion, and also
connotes that the client is hemodynamically stable, therefore, the repair is successful.

3. Answer: A

Rationale:

In cardiac catheterization or coronary angiogram, a catheter is inserted into the heart via a vein to measure the
ventricular function. A dye is used to provide further assessment of the structure and motion of the heart. Thallium is a
radioactive isotope injected parenterally so that the scintillation camera can count the radioactive uptake. This is
observed when a physician requests for a Nuclear Cardiology test. When performing a Transesophageal Echocardiogram
(TEE), a probe with a transducer tip is swallowed by the client to visualize for valvular abnormalities, possible thrombus,
bacterial endocarditis and any congenital heart defects. When performing an Intracoronary Ultrasound, a tiny ultrasound
probe is inserted into the coronary artery to evaluate for plaque size and consistency, arterial walls and effectiveness of
the treatment.

4. Answer: D

Rationale:

The central venous pressure is within the superior vena cava. The Normal CVP is 2-6 mmHg. A decrease in the CVP
indicates a decrease in the circulating blood volume, which may be a result of hemorrhage or fluid imbalances. The right
atrium is located at the midaxillary line at the fourth intercostal space, and the zero point on the transducer needs to be
at the level of the right atrium. The client needs to be supine, with the head of the bed elevated at 45 degrees to
correctly assess the CVP.

5. Answer: D

Rationale:
In hypokalemia, the electrocardiogram may show flattening and inversion of the T wave, the appearance of a U wave,
and ST depression. Hypercalcemia can cause a shortened ST and widened T wave. The electrocardiogram of a
hyperkalemic client shows tall peak T waves, widened QRS complexes, prolonged PR intervals or flat P waves.

6. Answer: A, B, E

Rationale:

Loading of drug depends on the type of dry powder inhaler. Take note that some dry powder inhalers do not require
loading. Dry powder inhalers are kept dry always and are place at room temperature. Never shake a dry powder inhaler.
It is not a pressurized container. There is no propellant, only the client’s breath can pull the drug in. Because the drug is a
dry powder and there is no propellant, the client will not feel, smell, or taste the drug during inhalation. The client’s
breath will moisten the powder causing it to clump and not be delivered accurately. Immediately after inhalation of drug,
the inhaler must be removed from the client’s mouth to prevent moisture.

7. Answer: C, D, E

Rationale:

Stripping is not allowed. Also when changing the drainage system or when checking air leaks, clamp the chest tube for
short periods only. Emptying of collection chamber or changing the drainage system should be done before the drainage
comes in contact with the bottom of the tube. Avoid kinks and dependent loops to allow effective drainage and prevent
disrupting the system. Report excessive drainage that is cloudy or red. Drainage will often increase with position changes
or coughing.

Bubbling is expected and indicates air drainage from the client. Absence of bubbling may mean that the chest tube is
obstructed, the lungs have fully reexpanded and no more air is leaking into the pleural space. Keep drainage system
lower than the level of the chest to allow effective drainage.

8. Answer: D

Rationale:

Sodium Bicarbonate should be slowly administered because fast infusion may result to abrupt reduction of serum
potassium level which can eventually lead to arrhythmias. Diluting or mixing the drug with hypotonic solution (i.e. D5W)
or isotonic solution (0.9% NaCl) can be ordered but not with hypertonic solution (I.e D10W).

9. Answer: D

Rationale:

Loosening tight clothing, applying warm blanket to prevent heat loss, and administering oxygen supply might help but is
not the priority this time. Direct pressure over the bleeding site is a priority to prevent shock but placing a tourniquet on
the artery is done by a surgeon. Because the client is showing signs of altered mental status, there is likely less perfusion
in the brain, which calls for fluid resuscitation. At least two IV access allows administration of fluids – crystalloid, blood or
clotting factors as necessary which is vital in correcting acidosis, hypothermia and coagulopathy, and to restore perfusion
rapidly.

10. Answer: A

Rationale:

Cushing’s syndrome is clinically defined as the presence of excessive corticosteroids. Addison’s disease is clinically
defined as adrenocortical insufficiency. Hypothyroidism is a condition wherein there is insufficient thyroid hormone
produced by the thyroid glands while SIADH is characterized by excessive release of anti-diuretic hormone.

11. Answer: C
Rationale:

Facial tremors will occur even without performing a specific maneuver. Exaggerated reflexes such as hyperreflexia can be
assessed by performing a different maneuver. Gentle tapping of the area below the zygomatic bone just in front of the
ear is used to elicit Chvostek sign to assess the presence of hypocalcemia. Trousseau sign is characterized by spasm of
the muscles of the hand and forearm upon inflation of a BP cuff on it.

12. Answer: A, B, C, D, E and F

Rationale:

The leaking of vesicant drugs into surrounding tissue causes local tissue damage like delayed healing, tissue necrosis,
disfigurement, loss of function and even amputation.

13. Answer: B, C, E

Rationale:

Manual stimulation is recommended to evacuate impacted stool. Having the client sit up straight and raise his head so
that he is looking ahead helps reduce the blood pressure as it allows gravitational pooling of blood in the lower
extremities. Constrictive clothing may trigger an autonomic reaction that would cause the blood pressure to go up so this
must be removed. Manual compression or tapping the bladder to allow urine to flow down the catheter should be
avoided because this would trigger an increase in blood pressure. Administration of prescribed vasodilators is done to
reduce high blood pressure.

14. Answer: D

Rationale:

Option A is ataxic breathing. Option B is apneustic breathing. Option C is central neurogenic hyperventilation. Cheyne-
stokes breathing respirations are a pattern of breathing in which phases of hyperpnea regularly alternate with apnea in a
crescendo-decrescendo pattern.

15. Answer: D

Rationale:

The lowest possible score for any response is 1. If a client is unresponsive to painful stimuli, the score is 1. A score lower
than 8 indicates that the client is in a comatose state. The highest score for the GCS is 15. A score of 15 indicates an alert
and oriented person. A score of 3-8 indicates severe head injury.

16. Answer: D

Rationale:

Helping the client maintain an exercise program is a therapeutic intervention to maintain joint mobility and good body
alignment. This will also prevent venous stasis due to impaired mobility. Client encouragement will not only address the
physical aspect of the disease but the client’s emotions and self-esteem as well. The safety of the client with impaired
physical mobility should always be considered. Continuous physical activity is not recommended. There should be an
alternate period of activity and rest to prevent excessive fatigue.

17. Answer: B

Rationale:

Receptive Aphasia refers to the inability to understand spoken words but can freely express and verbalize. Expressive
Aphasia refers to the inability to speak and communicate formulated thoughts and sentiments. Global aphasia affects
both expressive ability and auditory comprehension. Apraxia is characterized by loss of the ability to perform activities
that a person is physically able and willing to do.

18. Answer: B

Rationale:

Graphesthesia is the ability to identify the writing on the skin even with the eyes closed. The client provides a verbal
response, identifying the figure that was drawn. Option A is a test for stereognosis. Option C is a test used to assess the
Romberg’s sign while option D is a test for Kernig’s sign.

19. Answer: D

Rationale:

Any deterioration of oxygen saturation may necessitate intubation. However, the priority this time is to maintain a patent
airway. Infusion of IV fluids and administration of antibiotics are expected nursing actions but not the top priority this
time. Completing vaccination at this time will not suffice or treat the underlying respiratory problem. The situation calls
for a curative management and not preventive measures. Airway closure is the top priority. Throat examination is
avoided as this increases the risk of laryngeal obstruction. Aggression or agitation can also compromise airway and
breathing.

20. Answer: B

Rationale:

Absorption is the process when the drug is transferred from the site of origin into the bloodstream. Distribution occurs
when the drug in the blood is distributed to different parts of the body and accumulates in specific tissues. Metabolism
or biotransformation is the process wherein the drug is broken down into its inactive form. Excretion is the body’s
response to eliminate all the inactive form of the drug.

21. Answer: D

Rationale:

The Ventrogluteal site is safe for most intramuscular injections because it only involves the gluteus medius and gluteus
minimus muscles. The Vastus lateralis muscle is also a safe injection site for intramuscular medications because there are
no adjacent large blood vessels and nerves. The deltoid muscle is a smaller muscle and is safe for administration of
intramuscular medications less than 1 mL. The Dorsogluteal muscle is not recommended for intramuscular medications
because of the potential damage to the sciatic nerve. Large blood vessels are also located near the dorsogluteal muscle
and should be avoided.

22. Answer: B

Rationale:

Prescribing or stopping medications is the responsibility of the physician, thus the nurse must refer this first. Thiazide
diuretics cause loss of blood potassium while conserving blood calcium, thus, the electrolyte level must be evaluated
first.

23. Answer: D

The brandt-andrew maneuver is the proper extraction of the umbilical cord and placenta. McRobert’s maneuver is
performed in case of shoulder dystocia during childbirth. The Schultz mechanism is used to describe placental delivery.
The Ritgen’s maneuver is performed by applying pressure over the perineum and counter-pressure on the fetal head.
The Ritgen’s maneuver controls the exit of the fetal head and prevents severe damage to maternal tissues.
24. Answer: B

Rationale:

The uterine fundus should start to descend after 24 hours of delivery. The normal rate of uterine descent is 1 cm/day.

25. Answer: B

Rationale:

In Somatoform Disorder, there is no real organ damage, but the client verbalizes symptoms of a disease in an
unconscious manner. In Malingering, verbalization of symptoms of a disease is conscious and is used by the client to
achieve a secondary gain or benefit. Anxiety comes in many forms panic attacks, phobia, and social anxiety and the
distinction between a disorder and “normal” anxiety isn’t always clear. Amnesia refers to the loss of memories, such as
facts, information and experiences.

26. Answer: D

Rationale:

Not all chemotherapeutic agents alter the molecular structure of DNA. Chemotherapy should slow down cell division not
hasten it. All cells are sensitive to drug toxins. Chemotherapeutic agents act on all rapidly dividing cells – most action of
chemotherapeutic agents is that it affects all rapidly dividing cells including the normal and cancer cells.

27. Answer: C

Rationale:

Ketoacidosis is associated with high levels of ketone bodies in the body brought by breakdown of fatty acids and is not
related to vomiting. Metabolic acidosis happens when the body produces excessive quantity of acid. Severe vomiting will
result to loss of HCL and acids coming from extracellular fluids which in turn lead to metabolic alkalosis. Respiratory
alkalosis occurs when there is an increased respiration which elevates the blood pH beyond the normal range of 7.35-
7.45.

28. Answer: D

Rationale:

2nd degree skin reactions are evident by scaly skin, an itchy feeling and dry desquamation. Reddening of the skin is not
seen in 2nd degree skin reaction.

29. Answer: B

Rationale:

Tremor is clinically defined as the rhythmic and repetitive muscle movement. Chorea is clinically defined as brief and
involuntary muscle twitching of the face or limbs which hinders the client’s mobility. Athetosis is clinically defined as the
presence of irregular and slow twisting motions. Dystonia is similar to the definition of Athetosis but involves larger
muscle areas.

30. Answer: C

Rationale:

Standing close to the working area is a proper body mechanic to prevent muscle fatigue. The nurse should adjust the bed
to waist level in order to prevent stretching and muscle strain. Proper body mechanic includes turning the body as a
whole unit when moving the client to avoid twisting the back. The knees are bent to support the body’s center of gravity
and maintain body balance. Bending the knees will provide a wider base of support for effective leverage and use of
energy.

Potrebbero piacerti anche